Производные функции примеры и решения: Примеры решения производных с ответами

Содержание

Примеры решения производных

Используй поиск, чтобы найти научные материалы и собрать список литературы

База статей справочника включает в себя статьи написанные экспертами Автор24, статьи из научных журналов и примеры студенческих работ из различных вузов страны

Содержание статьи

1. Вычисление производных графически

2. Вычисление производных по смыслу

3. Вычисление табличных производных

Вычисление производных графически

Пример 1

На рисунке 1 изображен график функции и касательной к графику в точке с абсциссой x0. Найти значение производной функции в абсциссе.

Решение. Производная в точке равна отношению~приращения функции к приращению аргумента. Выберем на касательной две точки с целочисленными координатами. Пусть, например, это будут точки F (-2,0) и C (-3,2).

Рисунок 1. График функции

Найдем приращение аргумента:

$\Delta $x = х2 — х1 = -3 — (-2) = -1

Найдем приращение функции:

$\Delta $y = y2 — y1 = 2 — 0 = 2

Тогда значение производной:

\[\frac{\Delta y}{\Delta x} =-\frac{2}{1} =-2\]

Пример 2

На рисунке 2 изображен график функции. {2} x} \]

Сообщество экспертов Автор24

Автор этой статьи Дата последнего обновления статьи: 10.12.2021

Выполнение любых типов работ по математике

Решение задач по комбинаторике на заказ Решение задачи Коши онлайн Математика для заочников Контрольная работа на тему числовые неравенства и их свойства Контрольная работа на тему умножение и деление рациональных чисел Контрольная работа на тему действия с рациональными числами Дипломная работа на тему числа Курсовая работа на тему дифференциальные уравнения Контрольная работа на тему приближенные вычисления Решение задач с инвариантами

Подбор готовых материалов по теме

Дипломные работы Курсовые работы Выпускные квалификационные работы Рефераты Сочинения Доклады Эссе Отчеты по практике Решения задач Контрольные работы

Задания для самостоятельного решения

Понятие производной.

Правила дифференцирования. Таблица производных

Пусть определена в точке и в некоторой ее окрестности.

Пусть точка рассматриваемой окрестности, то приращением аргумента в точке называется величина , приращением функции – величина . Если выразить , то .

Производной функции в точке называется предел отношения приращения функции к приращению аргумента, когда последнее стремится к нулю, при условии, что предел существует.

Производную в точке обозначают . По определению

, (1)

или, что то же,

, (2)

при условии, что пределы (1),(2) существуют.

Функция, имеющая производную в точке, называется

дифференцируемой в этой точке. Операция нахождения производной называется дифференцированием.

Производная функции в точке – это число. Если функция дифференцируема на некотором множестве X из ее области определения, то также является функцией (ее обозначают также ).

 

Основные правила дифференцирования

Пусть -дифференцируемые функции. Справедливы формулы:

; (3)

; (4)

; (5)

; (6)

. (7)

 

Таблица производных основных элементарных функций

1) ,где ,

в частности

а) ,

б) ;

2) где ,

в частности

;

3) где ,

в частности

;

4) ;

5) ;

6) ;

7) ;

8) ;

9) ;

10) ;

11) ;

12) ;

13) ;

14) ;

15) .

Пример 1: Найти производную функции в точке , пользуясь определением, если:

1) , ;

2) .

Решение. 1.Используем определение производной в виде формулы (1):

Поскольку по условию , то

2. По формуле (1) получаем

Далее, применив тригонометрическую формулу , получим:

Так как при имеем и, применив формулу первого замечательного предела, получаем:

Поскольку по условию , то

Пример 2:Вычислить производную функции , пользуясь определением производной.

Решение.Пусть произвольная фиксированная точка из . Пользуясь формулой (1), имеем:

Таким образом, операция дифференцирования ставит в соответствие функции , функцию .

Пример 3.Найти производную функции:

1) ;

2) ; 3) .

Решение. 1. Дифференцируем функцию и используем формулы (4), (5) и таблицу производных, получаем:

2. Дифференцируем функцию по формулам (3), (4), (6) и соответствующим формулам таблицы производных:

3. Дифференцируем функцию по формулам (7), (5), (3) и первой формуле таблицы производных:

Пример 4. Вычислить производную функции, используя правила дифференцирования и таблицу производных:

1) 2) ;

3)

Решение.1. Преобразуем функцию, пользуясь свойствами логарифма:

Полученное выражение дифференцируем по формулам (4), (5), (6) и формулам таблицы производных:

 

2. Перед дифференцированием преобразуем выражение, пользуясь свойствами логарифма:

Дальше воспользуемся формулами (3), (4), (5) и таблицей производных:

3. Так как непосредственное дифференцирование вызывает значительные трудности, предварительно упростим выражение по формулам тригонометрии:

Полученное выражение дифференцируем по формуле (7) и соответствующим формулам таблицы производных.

Задания для самостоятельного решения

 

I уровень

1.1.Пользуясь определением, найдите производную функции:

1) 2)

1.2.Найдите производную функции:

1) ; 2) ;

3) ; 4) ;

5) ; 6) ;

7) ; 8) .

1.3. Найдите , если

1) ; 2) ;

3) ; 4) ;

5) ; 6) .

1.4.Вычислите:

1) , если: ;

2) если

;

3) если .

1.5. Вычислите , если

1.6.Вычислите , если .

1.7. Решите уравнение:

1) , где

2) , где .

 

II уровень

2.1. Найдите производные , предварительно преобразовав выражение:

1) ; 2) ;

3) ; 4) .

2.2. Для функции найдите

2.

3.Известно, что . Найдите .

2.4. Решите неравенство , где .

 

III уровень

 

3.1. Вычислите , если:

1) ,

2) , .

3.2. Пользуясь определением производной, найдите , где

3.3. Найдите значение производной функции в точке , если .

3.4.Найдите сумму значений производной функции в точках x = 1 и x = 0.

 

 



Дата добавления: 2016-07-27; просмотров: 6889; ЗАКАЗАТЬ НАПИСАНИЕ РАБОТЫ


Производная и ее применения – презентация онлайн

Производная и ее применения
Определение. Производной функции y=f(x),
заданной на некотором интервале (a;b), в точке х
этого интервала, называют предел отношения
приращения функции в этой точке к
соответствующему приращению аргумента,
когда приращение аргумента стремится к нулю.
Производную функции f(x) обозначают f ‘(x) и
говорят: «эф штрих от икс». Следовательно,
f
f ( x) lim
x x

3. Алгоритм нахождения производной (для функции y=f(x)).

Зафиксировать значение х, найти f(x).
Дать аргументу х приращение ∆х, перейти в
новую точку х+∆х, найти f(x+∆x).
Найти приращение функции: ∆у=f(x+∆x)–f(x).
у
Составим отношения
.
x
f
Вычислить lim x
x
Этот предел и есть f ‘(x).

4. Пример. Найти производную функции у=2х+3 в точке х=3

у (3) 2 3 3 9
f ( x x) 2 (3 x) 3 2 x 9
y y (3 x) y (3) 2 x
y 2 x
2
x x
f
2
lim
x x
у (3) = 2

5. Физический смысл производной

Если при прямолинейном движении путь s,
пройденной точкой, есть функция от времени t,
т.е. s=f(t), то скорость точки есть производная от
пути по времени, т.е. v(t)=f ‘(t), этот факт
выражает механический смысл производной.

6. пример

Тело движется по прямой так, что расстояние S (в
метрах) от него до точки В этой прямой изменяется
по закону S (t ) 2t 12t 3(t – время движения в
секундах). Через сколько секунд после начала
движения ускорение тела будет равно 36 м/ с 2 ?
Решение. Из механического смысла производной
имеем скорость – это производная пути по времени.
Скорость изменяется по закону v(t ) S (t ) 6t 24t . Так
как ускорение – это производная скорости по
времени, то ускорение изменяется по закону
a(t ) v (t ) 12t 24 , с другой стороны ускорение равно
2
с
36 м/ . Решим уравнение 12t 24 36
, t=5 c.
Ответ: через 5 секунд.
3
2
2

7. Геометрический смысл производной

Если в точке х 0 к графику функции y=f(x)
проведена касательная, то число f ‘( х0) есть
тангенс угла альфа между этой касательной и
положительным направлением оси ОХ, т.е.
f ‘( х 0)=tgα. Этот угол называю углом наклона
касательной. Этот факт выражает
геометрический смысл производной.

8. Пример

На рисунке изображен график функции y=f(x)
и касательная к нему в точке с абсциссой х .
Найдите значение производной функции f(x)
0
в точке х 0 .
Рис.1

9. Решение.

Значение производной f(x) в точке х 0 есть
значение тангенса угла, образованного
касательной к графику функции с
положительным направлением оси ОХ. Из
треугольника АВС (рис.1).
tga tg ( CAB)
Ответ: 1,75.
CB 7
1,75
AB 4

10. Вычисление производных

Формулами дифференцирования обычно
называют формулы для нахождения
производных конкретных функций.

11. Формулы дифференцирования

C 0
x 1
( x n ) n x n 1
1
( x )
2 x
1
1
2
x
x
(sin x) cos x
(cos x) sin x
1
cos 2 x
1
(ctgx) 2
sin x
arcsin x 1 2 , x 1
1 x
1
(arccos x)
, x 1
2
1 x
1
(arctgx)
1 x2
1
(arcctgx)
1 x2
(tgx)
(ln x)
1
x
1
(log a x)
x ln a
(e x ) e x
(a x ) a x ln a

12. Формулы дифференцирования

1
u
u
1
(log a u )
u
u ln a
(eu ) eu u
(ln u )
(a u ) a u ln a u

13.

Правила дифференцированияТеорема 1.
Если функции y=f(x) и y=g(x) имеют
производную в точке x, то и их сумма имеет
производную в точке x, причем производная
суммы равна сумме производных:
( f ( x) g ( x)) f x g ( x)

14. Теорема 2

Если функция y=f(x) имеет производную в точке
х, то и функция y=kf(x) имеет производную в
точке х, причем
kf x
kf x

15. Теорема 3

. Если функции y=f(x) и y=g(x) имеют
производную в точке x, то и их произведение
имеет производную в точке x, причем
( f ( x) g ( x)) f x g ( x) f ( x) g ( x)

16. Теорема 4

Если функции y=f(x) и y=g(x) имеют производную
в точке x и в этой точке g(x) ≠0,
то функция
y
f x
g x
имеет производную в точке х, причем
f x
f x g x f x g x
g 2 ( x)
g x

17. Теорема 5

Если функция f имеет производную в точке
х0
а функция g имеет производную в точке y0 f x0 ,
то сложная функция h x g f x также имеет
производную в точке х0 , причем
h x0 g f x0 f x0
Примеры. Найти
производные функций
.
Решения
1.
f x x 3 x 4 ;
1.
f x 3×3 1 4 x 4 1 3x 2 4 x3 .
2.
f x 3x 3 2 x 2 ;
2.
f x 3 3×3 1 2 2 x 2 1 9 x 2 4 x.
3.
f x x 2 x 3 x ;
3.
f x
x 2x
3
1
x x 2 x3 x
2 x 3 x x 2 3x 3 1 1
2 x
2 x3 x
x 6×2 1 .
2 x
4.
2×2 4
f x
;
5x 8
4.
2x
f x
2
4 5x 8 2 x 2 4 5 x 8
5x 8
2
5x 8
2
20 x 2 32 x 10 x 2 20 10 x 2 32 x 20
.
5x 8 2
5x 8 2
5.
f x 8 x 4 ;
3
4 x 5 x 8 2 x 2 4 5
2
2
2
5. f x 3 8 x 4 8 x 4 3 8 x 4 8 24 8 x 4 .

19. Применение производной при исследовании функции

Пример 1.
Функция y=f(x) определена на промежутке (-5;9). На
рисунке 2 изображен график производной этой
функции. Определите число касательных к графику
функции y=f(x), которые наклонены под углом 45 0
к положительному направлению оси абсцисс.
Рис.2

20. Решение.

Пусть α –угол касательной, проведенной к
графику функции y=f(x) в точке х 0

положительным направлением оси абсцисс,
тогда tg f x0 .
У=1
Рис.3
Так как tg 450 1 , то для
решения задачи
достаточно определить
количество точек
пересечения графика
функции y f x и
прямой у=1. Таких
точек четыре.

21. Пример 2

На рисунке 2 Рис.2 изображен график
производной функции y=f(x) найдите абсциссу
точки, в которой касательная к графику y=f(x)
параллельна прямой у=1 или совпадает с ней.
Решение. Так как касательная параллельна
прямой у=1, то ее угловой коэффициент равен 0
и тогда производная равна 0. По графику (рис.2)
определяем, что производная обращается в
ноль при х=-4; х=-0,5; х=3; х=7.
Рис.4
Ответ: -4; -0,5; 3; 7.

23. Пример 3.

На рисунке 5 изображен график функции y=f(x),
определенной на промежутке . Определите
количество целых точек, в которых
производная функции f(x) положительна.
Рис.5

24. Решение.

Производная функции
положительна в тех целых
точках, которые
принадлежат какомунибудь промежутку
возрастания, за
исключением точек, в
которых производная равна
нулю (в этих точках
касательная к графику
функции параллельна оси
ОХ) или не существует. По
рисунку 2 определяем
абсциссы таких точек: -4; -3;
2; 3; 4. Таких точек пять.
Рис.6

25. Пример 4.

На рисунке 5
изображен график
функции y=f(x), определенной на интервале (5;9). Определите количество целых точек, в
которых производная функции отрицательна.
Решение. Производная функции отрицательна в
тех целых точках, которые принадлежат
какому-нибудь промежутку убывания функции,
за исключением точек, в которых производная
равна нулю (в этих точках касательная к
графику функции параллельна оси ОХ) или не
существует. По рисунку определяем абсциссы
таких точек: -1; 0; 6; 7; 8. Таких точек пять.
Рис. 5
Рис.7
Ответ:5

27. Пример 5.

На рисунке 2
изображен график
производной функции y=f(x), определенной на
интервале (-5;9). Найдите промежутки
возрастания функции y=f(x). В ответе укажите
длину наибольшего из них.
Решение. Промежуткам возрастания функции
соответствуют промежутки, на которых
производная данной функции положительна.
По графику определяем, что наибольший из
этих промежутков имеет длину 4.
4
Рис.8

29. Пример 6.

На рисунке 2
изображен график
производной функции y=f(x), определенной на
интервале (-5;9). Найдите промежутки
убывания функции y=f(x). В ответе укажите
длину наибольшего из них.
Решение. Промежуткам убывания функции
соответствуют промежутки, на которых
производная данной функции отрицательна. По
графику определяем, что наибольший из этих
промежутков имеет длину 3,5.
Рис.2
3,5
Рис.9

31. Пример 7.

На рисунке Рис.2 изображен график
производной функции y=f(x), определенной на
интервале (-5;9). Найдите количество точек
максимума функции y=f(x).
Решение. Точек максимума здесь две, так как
график производной 4 раза меняет знак на
интервале (-5;9), из них два раза с плюса на
минус. Это и есть точки максимума.
Рис.10

33. Пример 8.

На рисунке
изображен график
производной функции y=f(x), определенной на
интервале (-5;9). Найдите точки минимума
функции y=f(x).
Решение. На графике производной видно, что на
интервале (-5;9)производная 4 раза меняет знак
в точках х=-4; х=-0,5; х=3; х=7. Причем в точках
х=-4; х=3 он меняется с минуса на плюс. Значит,
эти точки являются точками минимума, так как
в точках х=-4 и х=3 характер монотонности
функции f(x) меняется с убывания на
возрастание.
Рис.2
-4
3
Рис.11

35. Пример 9.

На рисунке
изображен график
производной функции y=f(x), определенной на
интервале (-5;9). Найдите количество точек
экстремума функции y=f(x).
Рис. 2
Решение. На промежутке (-5;9) точек
экстремума функции y=f(x) ровно четыре: -4; 0,5; 3; 7.
-3
-0,5
Рис.12
3
7

37. Пример 10

На рисунке 13 изображен график производной
функции y=f(x), определенной на интервале (5;4). Укажите абсциссы точек, в которой
касательная к графику функции y=f(x) имеет
наименьший и наибольший угловой
коэффициент.
Рис.13

38. Решение.

Угловой коэффициент касательной kкас. f x0 . По
графику определяем, что наименьшее значение
функция y f x достигает при x0 1 . А наибольшее
значение функция y f x достигает при x0 2 .
-2
Рис.14

39. Пример 11.

На рисунке
изображен график
производной функции y=f(x), определенной на
интервале (-5;9). Найдите количество точек, в
которых касательная к графику функции y=f(x)
параллельна прямой у=-4х+3 или совпадает с
ней.
Касательная к графику функции y=f(x) в
некоторой точке параллельна прямой у=-4х+3,
если значение производной функции в этой
точке равно угловому коэффициенту прямой, то
есть f x 4 . По графику (рис. 15) видно, что
принимает значение -4 в одной точке.
Рис.2
у=-4
Рис.15

41. Пример 12.

.
К графику функции y=f(x) проведена
касательная в точке с абсциссой x0 4 . На
рисунке 16 изображен график производной
этой функции. Определите градусную меру угла
наклона касательной.
Решение. Пусть
– угол наклона данной
касательной к оси абсцисс. Так как
.
,
то
f 4 1
tg 1
Отсюда получаем
.
Ответ: 1350
135 0
Рис. 16

42. Пример 14.

На
Рис.5
изображен график функции y=f(x),
определенной на промежутке (-5;9). Найдите
количество точек, в которых касательная к
графику функции параллельна прямой y=-7.
Решение. Так как касательные параллельны
прямой у=-7, то они параллельны оси ОХ,
следовательно, производные функции f(x) в
точках касания должны ровняться нулю. Это
стационарные точки. На рисунке все они
являются точками экстремума (максимумами
или минимумами). Их три.
Рис.17
Ответ: 3.
Рис.5

Вычисление производных степенно-показательных функций. Производная степенно-показательной функции

Содержание

  1. Таблица производных и правила дифференцирования
  2. Производные сложных функций
  3. Производные элементарных функций
  4. Геометрический и физический смысл производной
  5. Производная функции с экспонентой
  6. Понятие производной сложной функции
  7. Определение и формулы
  8. Пошаговые примеры – как найти производную
  9. Примеры решения задач
  10. Применение дифференциала в приближенных вычислениях
  11. Производная суммы и разности
  12. Таблица производных часто встречающихся функций
  13. Найти производные самостоятельно, а затем посмотреть решения
  14. Производная функции с корнем
  15. Логарифмическая производная
  16. Производная сложной степенной функции
  17. Правила нахождения производных
  18. Таблица производных некоторых сложных функций
  19. Примеры
  20. Производная произведения
  21. Продолжаем искать производные вместе
  22. Синтаксис описания формул
  23. Производная дроби с переменной произвольной степени в знаменателе
  24. 1-ый способ
  25. Производные простых функций
  26. Вычисление производной

Таблица производных и правила дифференцирования

Основные ссылки – таблица производных, правила дифференцирования и примеры решений (10 шт).

Пример

Задание. Найти производную функции

Решение. Так как производная суммы равна сумме производных, то

Воспользуемся формулами для производных показательной и обратной тригонометрической функций:

Ответ.

Производные сложных функций

Основные ссылки – теоретический материал и примеры решений (10 шт).

Пример

Задание.Найти производную функции

Решение. По правилу дифференцирования сложной функции:

В свою очередь производная также берется по правилу дифференцирования сложной функции:

Ответ.

Производные элементарных функций

Элементарные функции — это все, что перечислено ниже. Производные этих функций надо знать наизусть. Тем более что заучить их совсем несложно — на то они и элементарные.

Итак, производные элементарных функций:

НазваниеФункцияПроизводная
Константаf(x) = C, C ∈ R0 (да-да, ноль!)
Степень с рациональным показателемf(x) = x n n · x n − 1
Синусf(x) = sin xcos x
Косинусf(x) = cos x− sin x (минус синус)
Тангенсf(x) = tg x1/cos2 x
Котангенсf(x) = ctg x− 1/sin2 x
Натуральный логарифмf(x) = ln x1/x
Произвольный логарифмf(x) = log a x1/(x · ln a)
Показательная функцияf(x) = e x e x (ничего не изменилось)

Если элементарную функцию умножить на произвольную постоянную, то производная новой функции тоже легко считается:

(C · f)’ = C · f ’.

В общем, константы можно выносить за знак производной. Например:

(2x 3)’ = 2 · (x 3)’ = 2 · 3x 2 = 6x 2.

Очевидно, элементарные функции можно складывать друг с другом, умножать, делить — и многое другое. Так появятся новые функции, уже не особо элементарные, но тоже дифференцируемые по определенным правилам.

Геометрический и физический смысл производной

Пусть есть функция f(x), заданная в некотором интервале (a, b). Точки х и х0 принадлежат этому интервалу. При изменении х меняется и сама функция. Изменение аргумента – разность его значений х-х0. Эта разность записывается как дельта икс и называется приращением аргумента. Изменением или приращением функции называется разность значений функции в двух точках. Определение производной:

Производная функции в точке – предел отношения приращения функции в данной точке к приращению аргумента, когда последнее стремится к нулю.

Иначе это можно записать так:

Какой смысл в нахождении такого предела? А вот какой:

Геометрический смысл производной: производная от функции в точке равна тангенсу угла между осью OX и касательной к графику функции в данной точке.

Физический смысл производной: производная пути по времени равна скорости прямолинейного движения.

Действительно, еще со школьных времен всем известно, что скорость – это частное пути x=f(t) и времени t. Средняя скорость за некоторый промежуток времени:

Чтобы узнать скорость движения в момент времени t0 нужно вычислить предел:

Приведем пример, иллюстрирующий практическое применение производной. Пусть тело движется то закону:

Нам нужно найти скорость в момент времени t=2c. Вычислим производную:

Производная функции с экспонентой

производная сложной функции примеры, производная показательной функции

Тут на самом деле 2 функции: экспонента и степенная функция (с отрицательным показателем -1). Отсюда и поэтапное вычисление этой производной. В заблуждение может ввести натуральный логарифм, но заметьте – натуральный логарифм от 5 все лишь число.

Понятие производной сложной функции

Пусть y сложная функция x, т.е. y = f(u), u = g(x), или

Если g(x) и f(u) – дифференцируемые функции своих аргументов соответственно в точках x и u = g(x), то сложная функция также дифференцируема в точке x и находится по формуле

Типичная ошибка при решении задач на производные – машинальное перенесение правил дифференцирования простых функций на сложные функции. Будем учиться избегать этой ошибки.

Посмотрите на формулу 9 в таблице производных. Исходная функция является функцией от функции, причём аргумент x является аргументом лишь второй функции, а вторая функция является аргументом первой функции, или, согласно более строгому определению – промежуточным аргументом по независимой переменной x.

А теперь посмотрите на картинку ниже, которая иллюстрирует решение задач на сложные производные по аналогии с простым примером из кулинарии – приготовлении запечёных яблок, фаршированных ягодами.

Итак, “яблоко” – это функция, аргументом которой является промежуточный аргумент, а промежуточный аргумент по независимой переменной x, в свою очередь, является “фаршем” (ягодами). Представим себе, что решая задачи на производные сложной функции, сначала помещаем яблоко с фаршем в особую (физико-математическую) духовку и устанавливаем режим 1. При таком режиме духовка воздействует только на “яблоко”, поскольку нужно, допустим, больше пропечь яблоко, а фарш из ягод оставить более сочным, то есть обрабатывать в другом режиме. Итак, в при режиме 1 обрабатывается яблоко, а фарш остаётся незатронутым, или, ближе к нашим задачам, находим производную функции лишь от промежуточного аргумента, то есть, “яблока”. Затем в духовке устанавливается режим 2, который воздействует только на фарш, иначе говоря, записываем производную функции, являющейся промежуточным аргументом по независимой переменной x. И, в конце концов, записываем произведение производной “яблока” и производной “фарша”. Можно подавать!

Пример 1.Найти производную функции

Сначала определим, где здесь “яблоко”, то есть функция по промежуточному аргументу u, а где “фарш”, то есть промежуточный аргумент u по независимой переменной x. Определяем: возведение в степень – это функция по промежуточному аргументу, то есть “яблоко”, а выражение в скобках (разность двух тригонометрических функций) – это промежуточный аргумент, то есть “фарш”.

Тогда

Далее по таблице производных (производная суммы или разности, производные синуса и косинуса) находим:

Требуемая в условии задачи производная (готовое “фаршированое яблоко”):

Нахождение производной сложной логарифмической функции имеет свои особенности, поэтому у нас есть и урок “Производная логарифмической функции”.

Пример 2.Найти производную функции

Неправильное решение:вычислять натуральный логарифм каждого слагаемого в скобках и искать сумму производных:

Правильное решение:опять определяем, где “яблоко”, а где “фарш”. Здесь натуральный логарифм от выражения в скобках – это “яблоко”, то есть функция по промежуточному аргументу u, а выражение в скобках – “фарш”, то есть промежуточный аргумент u по независимой переменной x.

Тогда (применяя формулу 14 из таблицы производных)

Во многих реальных задачах выражение с логарифмом бывает несколько сложнее, поэтому и есть урок “Производная логарифмической функции”.

Пример 3.Найти производную функции

Неправильное решение:

Правильное решение.В очередной раз определяем, где “яблоко”, а где “фарш”. Здесь косинус от выражения в скобках (формула 7 в таблице производных)- это “яблоко”, оно готовится в режиме 1, воздействующем только на него, а выражение в скобках (производная степени – номер 3 в таблице производных) – это “фарш”, он готовится при режиме 2, воздействующей только на него. И как всегда соединяем две производные знаком произведения. Результат:

Производная сложной логарифмической функции – частое задание на контрольных работах, поэтому настоятельно рекомендуем посетить урок “Производная логарифмической функции”.

Первые примеры были на сложные функции, в которых промежуточный аргумент по независимой переменной был простой функцией. Но в практических заданиях нередко требуется найти производную сложной функции, где промежуточный аргумент или сам является сложной функцией или содержит такую функцию. Что делать в таких случаях? Находить производные таких функций по таблицам и правилам дифференцирования. Когда найдена производная промежуточного аргумента, она просто подставляется в нужное место формулы. Ниже – два примера, как это делается.

Кроме того, полезно знать следующее. Если сложная функция может быть представлена в виде цепочки из трёх функций

,

то её производную следует находить как произведение производных каждой из этих функций:

.

Для решения многих ваших домашних заданий может потребоваться открыть в новых окнах пособия Действия со степенями и корнями и Действия с дробями.

Нет времени вникать в решение? Можно заказать работу!

К началу страницы

Пройти тест по теме Производная, дифференциал и их применение

Пример 4.Найти производную функции

Применяем правило дифференцирования сложной функции, не забывая, что в полученном произведении производных промежуточный аргумент по независимой переменной x не меняется:

Готовим второй сомножитель произведения и применяем правило дифференцирования суммы:

Второе слагаемое – корень, поэтому

Таким образом получили, что промежуточный аргумент, являющийся суммой, в качестве одного из слагаемых содержит сложную функцию: возведение в степень – сложная функция, а то, что возводится в степень – промежуточный аргумент по независимой переменной x.

Поэтому вновь применим правило дифференцирования сложной функции:

Степень первого сомножителя преобразуем в корень, а дифференцируя второй сомножитель, не забываем, что производная константы равна нулю:

Теперь можем найти производную промежуточного аргумента, нужного для вычисления требуемой в условии задачи производной сложной функции y:

Тогда

Пример 5.Найти производную функции

Сначала воспользуемся правилом дифференцирования суммы:

Получили сумму производных двух сложных функций. Находим первую из них:

Здесь возведение синуса в степень – сложная функция, а сам синус – промежуточный аргумент по независимой переменной x. Поэтому воспользуемся правилом дифференцирования сложной функции, попутно вынося множитель за скобки:

Теперь находим второе слагаемое из образующих производную функции y:

Здесь возведение косинуса в степень – сложная функция f[g(x)], а сам косинус – промежуточный аргумент по независимой переменной x. Снова воспользуемся правилом дифференцирования сложной функции:

Результат – требуемая производная:

Определение и формулы

Степенно-показательная функция
– это функция, имеющая вид степенной функции
y = uv,
у которой основание u и показатель степени v являются некоторыми функциями от переменной x:
u = u(x) v = v(x).
Эту функцию также называют показательно-степенной или сложной показательной функцией.

Заметим, что степенно-показательную функцию можно представить в показательном виде:
.
Поэтому ее также называют сложной показательной функцией.

Далее мы покажем, что производная степенно-показательной функции определяется по формуле:
(1) .

Пошаговые примеры – как найти производную

Пример 3. Найти производную функции

.

Решение. Определяем части выражения функции: всё выражение представляет произведение, а его сомножители – суммы, во второй из которых одно из слагаемых содержит постоянный множитель. Применяем правило дифференцирования произведения: производная произведения двух функций равна сумме произведений каждой из этих функций на производную другой:

Далее применяем правило дифференцирования суммы: производная алгебраической суммы функций равна алгебраической сумме производных этих функций. В нашем случае в каждой сумме второе слагаемое со знаком минус. В каждой сумме видим и независимую переменную, производная которой равна единице, и константу (число), производная которой равна нулю. Итак, “икс” у нас превращается в единицу, а минус 5 – в ноль. Во втором выражении “икс” умножен на 2, так что двойку умножаем на ту же единицу как производную “икса”. Получаем следующие значения производных:

Подставляем найденные производные в сумму произведений и получаем требуемую условием задачи производную всей функции:

А проверить решение задачи на производную можно на калькуляторе производных онлайн.

Пример 4. Найти производную функции

Решение. От нас требуется найти производную частного. Применяем формулу дифференцирования частного: производная частного двух функций равна дроби, числитель которой есть разность произведений знаменателя на производную числителя и числителя на производную знаменателя, а знаменатель есть квадрат прежнего числителя. Получаем:

Производную сомножителей в числителе мы уже нашли в примере 2. Не забудем также, что произведение, являющееся вторым сомножителем в числителе в текущем примере берётся со знаком минус:

Если Вы ищете решения таких задач, в которых надо найти производную функции, где сплошное нагромождение корней и степеней, как, например, , то добро пожаловать на занятие “Производная суммы дробей со степенями и корнями”.

Если же Вам нужно узнать больше о производных синусов, косинусов, тангенсов и других тригонометрических функций, то есть, когда функция имеет вид вроде , то Вам на урок “Производные простых тригонометрических функций”.

Пример 5. Найти производную функции

Решение. В данной функции видим произведение, один из сомножителей которых – квадратный корень из независимой переменной, с производной которого мы ознакомились в таблице производных. По правилу дифференцирования произведения и табличному значению производной квадратного корня получаем:

Проверить решение задачи на производную можно на калькуляторе производных онлайн.

Пример 6. Найти производную функции

Решение. В данной функции видим частное, делимое которого – квадратный корень из независимой переменной. По правилу дифференцирования частного, которое мы повторили и применили в примере 4, и табличному значению производной квадратного корня получаем:

Чтобы избавиться от дроби в числителе, умножаем числитель и знаменатель на :

Проверить решение задачи на производную можно на калькуляторе производных онлайн.

Примеры решения задач

Применение дифференциала в приближенных вычислениях

Основные ссылки – теоретический материал и примеры решений (10 шт).

Задание. Вычислить приближенно , заменяя приращение функции ее дифференциалом.

Решение. Рассмотрим функцию . Необходимо вычислить ее значение в точке . Представим данное значение в виде следующей суммы:

Величины и выбираются так, чтобы в точке можно было бы достаточно легко вычислить значение функции и ее производной, а было бы достаточно малой величиной. С учетом этого, делаем вывод, что , то есть , .

Вычислим значение функции в точке :

Далее продифференцируем рассматриваемую функцию и найдем значение :

Тогда

Итак,

Ответ.

Производная суммы и разности

Пусть даны функции f(x) и g(x), производные которых нам известны. К примеру, можно взять элементарные функции, которые рассмотрены выше. Тогда можно найти производную суммы и разности этих функций:

  1. (f + g)’ = f ’ + g ’
  2. (f − g)’ = f ’ − g ’

Итак, производная суммы (разности) двух функций равна сумме (разности) производных. Слагаемых может быть больше. Например, (f + g + h)’ = f ’ + g ’ + h ’.

Строго говоря, в алгебре не существует понятия «вычитание». Есть понятие «отрицательный элемент». Поэтому разность f − g можно переписать как сумму f + (−1) · g, и тогда останется лишь одна формула — производная суммы.

Задача. Найти производные функций: f(x) = x 2 + sin x; g(x) = x 4 + 2x 2 − 3.

Функция f(x) — это сумма двух элементарных функций, поэтому:

f ’(x) = (x 2 + sin x)’ = (x 2)’ + (sin x)’ = 2x + cos x;

Аналогично рассуждаем для функции g(x). Только там уже три слагаемых (с точки зрения алгебры):

g ’(x) = (x 4 + 2x 2 − 3)’ = (x 4 + 2x 2 + (−3))’ = (x 4)’ + (2x 2)’ + (−3)’ = 4x 3 + 4x + 0 = 4x · (x 2 + 1).

Ответ:
f ’(x) = 2x + cos x;
g ’(x) = 4x · (x 2 + 1).

Таблица производных часто встречающихся функций

В следующей таблице приведены формулы для производных от степенных, показательных (экспоненциальных), логарифмических, тригонометрических и обратных тригонометрических функций. Доказательство большинства их этих формул выходит за рамки школьного курса математики.

ФункцияФормула для производнойНазвание формулы

y = c ,

где c – любое число

y’ = 0Производная от постоянной функции

y = x c ,

где c – любое число

y’ = c xc – 1Производная степенной функции
y = e xy’ = e xПроизводная от экспоненты (показательной функции с основанием e)

y = a x

где a – любое положительное число, не равное 1

y’ = a x ln aПроизводная от показательной функции с основанием a
y = ln x , x > 0, x > 0Производная от натурального логарифма

y = log a x , x > 0

где a – любое положительное число, не равное 1

, x > 0Производная от логарифма по основанию a
y = sin xy’ = cos xПроизводная синуса
y = cos xy’ = – sin xПроизводная косинуса

y = tg x ,

, ,Производная тангенса

y = ctg x ,

, ,Производная котангенса

y = arcsin x ,

Производная арксинуса

y = arccos x ,

Производная арккосинуса
y = arctg xПроизводная арктангенса
y = arcctg xПроизводная арккотангенса
Производная от постоянной функции

Функция:

y = c ,

где c – любое число

Формула для производной:

y’ = 0

Производная степенной функции

Функция:

y = x c ,

где c – любое число

Формула для производной:

y’ = c xc – 1

Производная от экспоненты (показательной функции с основанием e)

Функция:

y = e x

Формула для производной:

y’ = e x

Производная от показательной функции с основанием a

Функция:

y = a x

где a – любое положительное число, не равное 1

Формула для производной:

y’ = a x ln a

Производная от натурального логарифма

Функция:

y = ln x , x > 0

Формула для производной:

, x > 0

Производная от логарифма по основанию a

Функция:

y = log a x , x > 0

где a – любое положительное число, не равное 1

Формула для производной:

, x > 0

Производная синуса

Функция:

y = sin x

Формула для производной:

y’ = cos x

Производная косинуса

Функция:

y = cos x

Формула для производной:

y’ = – sin x

Производная тангенса

Функция:

y = tg x ,

где

Формула для производной:

,

Производная котангенса

Функция:

y = ctg x ,

где

Формула для производной:

,

Производная арксинуса

Функция:

y = arcsin x ,

Формула для производной:

Производная арккосинуса

Функция:

y = arccos x ,

Формула для производной:

Производная арктангенса

Функция:

y = arctg x

Формула для производной:

Производная арккотангенса

Функция:

y = arcctg x

Формула для производной:

Найти производные самостоятельно, а затем посмотреть решения

Пример 7. Найти производную функции

.

Правильное решение и ответ.

Пример 8. Найти производную функции

.

Правильное решение и ответ.

Пример 9. Найти производную функции

, где a и b – константы.

Правильное решение и ответ.

Пример 10. Найти производную функции

.

Правильное решение и ответ.

Пример 11. Найти производную функции

.

Правильное решение и ответ.

Производная функции с корнем

производная сложной функции примеры, производная функции с корнем

Степенная функция плюс осложнение внутри скобки. Пример не дорешан, всего лишь нужно вспомнить из таблицы пару элементарных функций.

Логарифмическая производная

Если производная от логарифмов – это такая сладкая музыка, то возникает вопрос, а нельзя ли в некоторых случаях организовать логарифм искусственно? Можно! И даже нужно.

Пример 11

Найти производную функции

Похожие примеры мы недавно рассмотрели. Что делать? Можно последовательно применить правило дифференцирования частного, а потом правило дифференцирования произведения. Недостаток способа состоит в том, что получится огромная трехэтажная дробь, с которой совсем не хочется иметь дела.

Но в теории и практике есть такая замечательная вещь, как логарифмическая производная. Логарифмы можно организовать искусственно, «навесив» их на обе части:

Примечание: т.к. функция может принимать отрицательные значения, то, вообще говоря, нужно использовать модули: , которые исчезнут в результате дифференцирования. Однако допустимо и текущее оформление, где по умолчанию принимаются во внимание комплексные значения. Но если со всей строгостью, то и в том и в другом случае следует сделать оговорку, что , которые исчезнут в результате дифференцирования. Однако допустимо и текущее оформление, где по умолчанию принимаются во внимание комплексные значения. Но если со всей строгостью, то и в том и в другом случае следует сделать оговорку, что .

Теперь нужно максимально «развалить» логарифм правой части (формулы перед глазами?). Я распишу этот процесс очень подробно:






Собственно приступаем к дифференцированию.
Заключаем под штрих обе части:

Производная правой части достаточно простая, её я комментировать не буду, поскольку если вы читаете этот текст, то должны уверенно с ней справиться.

Как быть с левой частью?

В левой части у нас сложная функция. Предвижу вопрос: «Почему, там же одна буковка «игрек» под логарифмом?».

Дело в том, что эта «одна буковка игрек» – САМА ПО СЕБЕ ЯВЛЯЕТСЯ ФУНКЦИЕЙ (если не очень понятно, обратитесь к статье Производная от функции, заданной неявно). Поэтому логарифм – это внешняя функция, а «игрек» – внутренняя функция. И мы используем правило дифференцирования сложной функции :

В левой части как по мановению волшебной палочки у нас «нарисовалась» производная . Далее по правилу пропорции перекидываем «игрек» из знаменателя левой части наверх правой части:

А теперь вспоминаем, о каком таком «игреке»-функции мы рассуждали при дифференцировании? Смотрим на условие:

Окончательный ответ:

Пример 12

Найти производную функции

Это пример для самостоятельного решения. Образец оформления примера данного типа в конце урока.

С помощью логарифмической производной можно было решить любой из примеров № 4-7, другое дело, что там функции проще, и, может быть, использование логарифмической производной не слишком-то и оправдано.

Производная сложной степенной функции

В сложной функции вместо x представлено более сложное выражение. Производная такой функции определяется по формуле:

(yn) = nyn-1 ⋅ y

Правила нахождения производных

Сам процесс нахождения производной называется дифференцированием. Функция, которая имеет производную в данной точке, называется дифференцируемой.

Как найти производную? Согласно определению, нужно составить отношение приращения функции и аргумента, а затем вычислить предел при стремящемся к нулю приращении аргумента. Конечно, можно вычислять все производные так, но на практике это слишком долгий путь. Все уже давно посчитано до нас. Ниже приведем таблицу с производными элементарных функций, а затем рассмотрим правила вычисления производных, в том числе и производных сложных функций с подробными примерами.

Таблица производных некоторых сложных функций

Для сложных функций на основании правила дифференцирования сложной функции формула производной простой функции принимает другой вид.

1. Производная сложной степенной функции, где u – дифференцируемая функция аргумента x
2. Производная корня от выражения
3. Производная показательной функции
4. Частный случай показательной функции
5. Производная логарифмической функции с произвольным положительным основанием а
6. Производная сложной логарифмической функции, где u – дифференцируемая функция аргумента x
7. Производная синуса
8. Производная косинуса
9. Производная тангенса
10. Производная котангенса
11. Производная арксинуса
12. Производная арккосинуса
13. Производная арктангенса
14. Производная арккотангенса

Примеры

Решим примеры. Преобразования, позволяющие применить другие свойства производной, мы применять не будем. В решениях будем использовать только формулу производной от дроби.

По условию даются функции. Нужно найти производные.

Пример 1


Рисунок 2. Пример. Автор24 — интернет-биржа студенческих работ


Рисунок 3. Пример. Автор24 — интернет-биржа студенческих работ


Рисунок 4. Пример. Автор24 — интернет-биржа студенческих работ

Производная произведения

Математика — наука логичная, поэтому многие считают, что если производная суммы равна сумме производных, то производная произведения strike“>равна произведению производных. А вот фиг вам! Производная произведения считается совсем по другой формуле. А именно:

(f · g) ’ = f ’ · g + f · g ’

Формула несложная, но ее часто забывают. И не только школьники, но и студенты. Результат — неправильно решенные задачи.

Задача. Найти производные функций: f(x) = x 3 · cos x; g(x) = (x 2 + 7x − 7) · e x .

Функция f(x) представляет собой произведение двух элементарных функций, поэтому все просто:

f ’(x) = (x 3 · cos x)’ = (x 3)’ · cos x + x 3 · (cos x)’ = 3x 2 · cos x + x 3 · (− sin x) = x 2 · (3cos x − x · sin x)

У функции g(x) первый множитель чуть посложней, но общая схема от этого не меняется. Очевидно, первый множитель функции g(x) представляет собой многочлен, и его производная — это производная суммы. Имеем:

g ’(x) = ((x 2 + 7x − 7) · e x )’ = (x 2 + 7x − 7)’ · e x + (x 2 + 7x − 7) · (e x )’ = (2x + 7) · e x + (x 2 + 7x − 7) · e x = e x · (2x + 7 + x 2 + 7x −7) = (x 2 + 9x) · e x = x(x + 9) · e x .

Ответ:
f ’(x) = x 2 · (3cos x − x · sin xg ’(x) = x(x + 9) · e x .

Обратите внимание, что на последнем шаге производная раскладывается на множители. Формально этого делать не нужно, однако большинство производных вычисляются не сами по себе, а чтобы исследовать функцию. А значит, дальше производная будет приравниваться к нулю, будут выясняться ее знаки и так далее. Для такого дела лучше иметь выражение, разложенное на множители.

Продолжаем искать производные вместе

Пример 12. Найти производную функции

.

Решение. Применяя правила вычисления производной алгебраической суммы функций, вынесения постоянного множителя за знак производной и формулу производной степени (в таблице производных – под номером 3), получим

.

Пример 13. Найти производную функции

Решение. Применим правило дифференцирования произведения, а затем найдём производные сомножителей, так же, как в предыдущей задаче, пользуясь формулой 3 из таблицы производных. Тогда получим

Пример 14. Найти производную функции

Решение. Как и в примерах 4 и 6, применим правило дифференцирования частного:

Теперь вычислим производные в числителе и перед нами уже требуемый результат:

Пример 15.Найти производную функции

Шаг1. Применяем правило дифференцирования суммы:

Шаг2. Найдём производную первого слагаемого. Это табличная производная квадратного корня (в таблице производных – номер 5):

Шаг3. В частном знаменатель – также корень, только не квадратный. Поэтому преобразуем этот корень в степень:

и далее дифференцируем частное, не забывая, что число 2 в первом слагаемом числителя – это константа, производная которой равна нулю, и, следовательно всё первое слагаемое равно нулю:

Корень из константы, как не трудно догадаться, является также константой, а производная константы, как мы знаем из таблицы производных, равна нулю:

,

а производная, требуемая в условии задачи:

Синтаксис описания формул

В описании функции допускается использование одной переменной (обозначается как x), скобок, числа пи (pi), экспоненты (e), математических операций: + — сложение, — вычитание, * — умножение, / — деление, ^ — возведение в степень.
Допускаются также следующие функции: sqrt — квадратный корень, exp — e в указанной степени, lb — логарифм по основанию 2, lg — логарифм по основанию 10, ln — натуральный логарифм (по основанию e), sin — синус, cos — косинус, tg — тангенс, ctg — котангенс, sec — секанс, cosec — косеканс, arcsin — арксинус, arccos — арккосинус, arctg — арктангенс, arcctg — арккотангенс, arcsec — арксеканс, arccosec — арккосеканс, versin — версинус, vercos — коверсинус, haversin — гаверсинус, exsec— экссеканс, excsc — экскосеканс, sh — гиперболический синус, ch — гиперболический косинус, th — гиперболический тангенс, cth — гиперболический котангенс, sech — гиперболический секанс, csch — гиперболический косеканс, abs — абсолютное значение (модуль), sgn — сигнум (знак), logP — логарифм по основанию P, например log7(x) — логарифм по основанию 7, rootP — корень степени P, например root3(x) — кубический корень.

Производная дроби с переменной произвольной степени в знаменателе

( 1 / xс )’ = – c / xс+1

Пример нахождения производной дроби:
( 1 / x2 )’ = – 2 / x3 .

(впереди ставим минус, показатель степени переменной поднимаем в числитель дроби, а степень переменной в знаменателе увеличиваем на единичку. Немного “ненаучно”, но подходит для быстрого запоминания)

1-ый способ

Применяя формулу:

То есть вначале производная берется как от степенной функции, а потом как от показательной.

Замечание

Порядок следования слагаемых неважен: можно вначале взять производную от показательной функции, а затем как от степенной, так как от перестановки слагаемых сумма не меняется:

Пример

Задание. Найти производную функции

Решение. Применяем формулу. В рассматриваемом случае

Тогда имеем:

Ответ.

Производные простых функций

1. Производная от числа равна нулю
с´ = 0
Пример:
5´ = 0

Пояснение:
Производная показывает скорость изменения значения функции при изменении аргумента. Поскольку число никак не меняется ни при каких условиях – скорость его изменения всегда равна нулю.
2. Производная переменной равна единице
x´ = 1

Пояснение:
При каждом приращении аргумента (х) на единицу значение функции (результата вычислений) увеличивается на эту же самую величину. Таким образом, скорость изменения значения функции y = x точно равна скорости изменения значения аргумента.
3. Производная переменной и множителя равна этому множителю
сx´ = с
Пример:
(3x)´ = 3
(2x)´ = 2
Пояснение:
В данном случае, при каждом изменении аргумента функции (х) ее значение (y) растет в с раз. Таким образом, скорость изменения значения функции по отношению к скорости изменения аргумента точно равно величине с.

Откуда следует, что
(cx + b)’ = c
то есть дифференциал линейной функции y=kx+b равен угловому коэффициенту наклона прямой (k).

4. Производная переменной по модулю равна частному этой переменной к ее модулю
|x|’ = x / |x| при условии, что х ≠ 0
Пояснение:
Поскольку производная переменной (см. формулу 2) равна единице, то производная модуля отличается лишь тем, что значение скорости изменения функции меняется на противоположное при пересечении точки начала координат (попробуйте нарисовать график функции y = |x| и убедитесь в этом сами. Именно такое значение и возвращает выражение x / |x| . Когда x < 0 оно равно (-1), а когда x > 0 – единице. То есть при отрицательных значениях переменной х при каждом увеличении изменении аргумента значение функции уменьшается на точно такое же значение, а при положительных – наоборот, возрастает, но точно на такое же значение.
5. Производная переменной в степени равна произведению числа этой степени и переменной в степени, уменьшенной на единицу
( xc )’= cxc-1, при условии, что xc и сxc-1,определены а с ≠ 0
Пример:
(x2 )’ = 2x
(x3)’ = 3x2
Для запоминания формулы:
Снесите степень переменной “вниз” как множитель, а потом уменьшите саму степень на единицу. Например, для x2 – двойка оказалась впереди икса, а потом уменьшенная степень (2-1=1) просто дала нам 2х. То же самое произошло для x3 – тройку “спускаем вниз”, уменьшаем ее на единицу и вместо куба имеем квадрат, то есть 3x2 . Немного “не научно”, но очень просто запомнить.
6. Производная дроби 1/х
(1/х)’ = – 1 / x2
Пример:
Поскольку дробь можно представить как возведение в отрицательную степень
(1/x)’ = (x-1 )’ , тогда можно применить формулу из правила 5 таблицы производных
(x-1 )’ = -1x-2 = – 1 / х2
7. Производная дроби с переменной произвольной степени в знаменателе
( 1 / xc )’ = – c / xc+1
Пример:
( 1 / x2 )’ = – 2 / x3
8. Производная корня (производная переменной под квадратным корнем)
( √x )’ = 1 / ( 2√x ) или 1/2 х-1/2
Пример:
( √x )’ = ( х1/2 )’ значит можно применить формулу из правила 5
( х1/2 )’ = 1/2 х-1/2 = 1 / (2√х)
9. Производная переменной под корнем произвольной степени
( n√x )’ = 1 / ( n n√xn-1 )
.

Вычисление производной

Вычисление производной — дело нехитрое, достаточно знать несколько простых правил и формулы дифференцирования простых функций; сложнее в этом онлайн калькуляторе было сделать интерпретатор математических выражений и алгоритм упрощения полученного результата, но об этом как-нибудь в другой раз…

Источники

  • https://www. webmath.ru/primeri_reshenii/derivative.php
  • https://www.berdov.com/docs/fluxion/rules/
  • https://Zaochnik.ru/blog/proizvodnaya-dlya-chajnikov-opredelenie-kak-najti-primery-reshenij/
  • https://zen.yandex.ru/media/maminov/proizvodnaia-slojnoi-funkcii-5bd5cda71ea36b00aa3a19ae
  • https://function-x.ru/derivative2.html
  • https://1cov-edu.ru/mat_analiz/proizvodnaya/nayti/logarifmicheskaya/stepenno-pokazatelnaya-funktsiya/
  • https://function-x.ru/derivative.html
  • http://ru.solverbook.com/spravochnik/proizvodnye/proizvodnaya-stepennoj-funkcii/
  • https://www.resolventa.ru/spr/matan/derivative_rule.htm
  • http://www.mathprofi.ru/slozhnye_proizvodnye_logarifmicheskaja_proizvodnaja.html
  • https://MicroExcel.ru/proizvodnaya-stepennoy-funktsii/
  • https://spravochnick.ru/matematika/formula_proizvodnoy_ot_drobi_primery/
  • https://planetcalc.ru/675/
  • https://profmeter.com.ua/communication/learning/course/course17/lesson642/
  • https://www. webmath.ru/poleznoe/formules_8_17.php
  • https://profmeter.com.ua/communication/learning/course/course17/lesson251/

Сообщество Экспонента

  • Публикация
  • 15.09.2022

Системы управления, Другое

Видел видос на канале экспоненты по созданию топливной системы. Вопрос заключается в наличии более полного описания готового примера или соответсвующее документации. Я новичок в симулинке и ещё многого не знаю. Адекватных и раскрытых пособий по созданию гидрав…

Моделирование гидравлических систем в simulink

  • Публикация
  • 10.09.2022

Системы управления, Электропривод и силовая электроника, Другое

Планирую написать книгу про модельно-ориентированное программирование с автоматическим генерированием кода применительно к разработке разнообразных микропроцессорных систем управления электроприводов. В этой книге в научно-практическо-методической форме я план…

Планирую написать книгу про модельно-ориентированное программирование с автоматическим генерированием кода применительно к разработке разнообразных микропроцессорных систем управления электроприводов.

  • Публикация
  • 24.08.2022

Цифровая обработка сигналов, Системы связи, Математика и статистика

                                                                          &…

Здесь собрана литература по комбинированным методам множественного доступа, в которых используется разделение пользователей в нескольких ресурсных пространствах.

  • вопрос
  • 23.08.2022

Математика и статистика, Радиолокация, Цифровая обработка сигналов

Есть записанный сигнал с датчика (синус с шумом). Как определить соотношение сигнал/шум?

Есть записанный сигнал с датчика (синус с шумом). Как определить соотношение сигнал/шум?

4 Ответа

  • ЦОС
  • цифровая обработка сигналов

23.08.2022

  • Публикация
  • 23.08.2022

Цифровая обработка сигналов, Системы связи, Математика и статистика

                                                                          &. ..

Здесь соборана литература по методам множественного доступа с поляризационным разделением и разделением по орбитальном угловому моменту.

  • Публикация
  • 16.08.2022

Цифровая обработка сигналов, Системы связи, Математика и статистика

                                      

Здесь собрана литература по методам множественного доступа с пространственным разделением.

  • вопрос
  • 22.07.2022

Изображения и видео, Цифровая обработка сигналов, Математика и статистика, Биология, Встраиваемые системы, Глубокое и машинное обучение(ИИ), Автоматизация испытаний, ПЛИС и СнК, Системы управления, Другое

Здравствуйте. Мне нужно обработать большое количество файлов с похожими названиями, каждый блок файлов относится к отдельному объекту, например: file_1_1.txt file_1_2.txt file_1_3.txt file_1_4.txt fil…

Здравствуйте. Мне нужно обработать большое количество файлов с похожими названиями, каждый блок файлов относится к отдельному объекту, например: file_1_1.txt file_1_2.txt file_1_3.txt file_1_4.txt fil…

2 Ответа

  • чтение

22.07.2022

  • вопрос
  • 17.07.2022

Математика и статистика, Цифровая обработка сигналов

Уважаемые коллеги, добрый вечер! В общем, возникла проблема следующего характера. Имеется сигнал, достаточно большой объем точек, длительность порядка 35-40 секунд. Он представлят собой последовательн…

Уважаемые коллеги, добрый вечер! В общем, возникла проблема следующего характера. Имеется сигнал, достаточно большой объем точек, длительность порядка 35-40 секунд. Он представлят собой последовательн…

  • MATLAB
  • Signal Processing

17.07.2022

  • вопрос
  • 15.07.2022

Системы связи, Цифровая обработка сигналов

Здравствуйте! Сделала в симулинке модель сигнала с модуляцией QPSK. На входе сигнала подала последоватльномть с Генератора Бернули бинарного. Sample time: 1/8000. ПРи выводе сигнала на анализатор спек…

Здравствуйте! Сделала в симулинке модель сигнала с модуляцией QPSK. На входе сигнала подала последоватльномть с Генератора Бернули бинарного. Sample time: 1/8000. ПРи выводе сигнала на анализатор спек…

  • сигнал
  • модуляция
  • qpsk
  • скорость бита
  • битрейт
  • символьная скорость
  • скорость передачи информации
  • цифровая манипуляция

15.07.2022

  • Публикация
  • 13.07.2022

Цифровая обработка сигналов, Системы связи, Математика и статистика

                                                                          &…

Здесь собрана литература по методам множественного доступа с кодовым разделением

Результаты поиска

Нет результатов поиска, попробуйте задать другие параметры.

Примеры вычисления основных производных. Типовые задачи 10 класс онлайн-подготовка на Ростелеком Лицей

Определение производной функции

 

Производная функции  в точке :

 

Геометрический смысл производной

Если задан график функции , то производная в точке  – это тангенс угла наклона касательной к данной функции в точке c абсциссой  (или угловой коэффициент касательной).

Физический смысл производной

Если в качестве функции мы берем перемещение, зависящее от , – , то , где  – перемещение,  – время, а  – мгновенная скорость в данной точке.

И сегодня мы попробуем вычислить некоторые производные по определению.

 

Производная функции

 

 

Начнем с самого простого – с линейной функции.

 

Пусть , где  и  – некоторые числа, а  – переменная.

Тогда:

Итак, выясняется, что  для любого . Значит, можно утверждать, что .

О чем это говорит?

Во-первых, мы подтвердили несколько фактов про линейную функцию, которые нам, возможно, уже были известны.

1. Так, исходя из геометрического смысла производной, тангенс угла наклона прямой совпадает с ее угловым коэффициентом (он равен производной в соответствующей точке).

Кроме этого, мы видим, что раз производная постоянна, то угол наклона постоянен, это вполне соответствует нашим представлениям о прямой.

2. Если предположить, что материальная точка движется прямолинейно равномерно, то ее координата в данный момент времени описывается функцией: , где  – начальная координата, а  – скорость. Рассмотрим это утверждение.

Предположим, что есть некоторая материальная точка, которая двигается по закону . Найти его производную.

Решение

Для удобства предположим, что точка движется равномерно, то есть  в каждой точке одинаково. Тогда с точки зрения физического смысла мы получим:

.

Физический смысл производной: производная от координаты равна мгновенной скорости точки в данный момент времени.

Но для равномерного движения мгновенная скорость в любой момент времени одна и та же и равна скорости движения тела . Получаем, что должно выполняться равенство: .

 

Следствия производной линейной функции

 

 

Во-первых, . Это следует из наших выкладок просто в силу того, что мы подставили ; .

 

Далее, . Итак, производная от константы равна нулю.

 

Производная функции

 

 

Дальше рассмотрим производную функции .

 

В силу того что  – произвольна, имеем:

Где это может нам пригодиться? В дальнейшем с помощью производных мы будем исследовать свойства функций, говорить об их монотонности и т.  д. Пока же мы можем говорить лишь о физическом и геометрическом смыслах. Разберем по примеру на каждый из них.

 

Пример 1

 

 

Тело движется по закону  ( – в секундах,  – в метрах). Какой будет скорость тела через 3 секунды после начала движения? Через какое время после начала движения скорость тела будет равна ?

 

Дано: , .

Найти: 1. , 2.

Прежде всего вспомним, что .

Отсюда мы можем вывести, что скорость через три секунды, то есть при , будет равна .

А  скорость будет равна через 5 секунд ().

Ответ: , .

В качестве небольшого упражнения попробуйте сами вывести производную функции .

А сделав это, вы сможете решать физические задачи на равноускоренное движение. , где  – ускорение.

 

Пример 2

 

 

В какой точке графика  его касательная параллельна прямой ?

 

Дано: , .

Решение

Раз прямые параллельны, то их угловые коэффициенты равны, то есть угловой коэффициент касательной (он же тангенс угла наклона) должен быть равен 1. Но мы помним, что тангенс угла наклона касательной как раз равен производной в точке касания (в абсциссе точки касания) (см. рис. 1).

Рис. 1. Иллюстрация к примеру 1

Приравнивая  имеем , значит, речь идет о касательной в точке .

Ответ: в точке .

 

Пример 3

 

 

Теперь рассмотрим кубическую функцию .

 

В силу того, что  произволен, имеем:

.

 

Итоги урока

 

 

На этом уроке мы с вами применяли определение производной на конкретных примерах. Мы вычислили производную линейной функции, производную  и , а также пока без доказательства познакомились с производной от . Кроме этого, мы разобрали несколько примеров, увидели, где применяется производная.

 

 

Доказательство

.

По формуле бинома Ньютона:

Это доказательство верно только для натуральных , так как бином Ньютона работает только для натуральных .

 

Список литературы

1. Алгебра и начала анализа, 10 класс (в двух частях). Учебник для общеобразовательных учреждений (профильный уровень) / под ред. А.Г. Мордковича. – М.: Мнемозина, 2009.

2. Алгебра и начала анализа, 10 класс (в двух частях). Задачник  для общеобразовательных учреждений (профильный уровень) / под ред. А.Г. Мордковича. – М.: Мнемозина, 2007.

3. Виленкин Н.Я., Ивашев-Мусатов О.С., Шварцбурд С.И. Алгебра и математический анализ для 10 класса (учебное пособие для учащихся школ и классов с углубленным изучением математики) – М.: Просвещение, 1996.

4. А.Н. Колмогоров, А.М. Абрамов, Ю.П. Дудницын и др. Алгебра и начала анализа. Учеб. для 10-11 кл. – М.: Просвещение, 1990.

 

Дополнительные рекомендованные ссылки на ресурсы сети Интернет

1. Интернет-сайт kontromat.ru (Источник)

2. Интернет-сайт YouTube (Источник)

3. Интернет-сайт cleverstudents.ru (Источник)

 

Домашнее задание

1. Вычислить производную функции .

2. Вычислить производную функции  в точке .

3. Найти производную функции .

 

3.2: Производная как функция

  1. Последнее обновление
  2. Сохранить как PDF
  • Идентификатор страницы
    2491
    • Гилберт Странг и Эдвин «Джед» Герман
    • OpenStax
    Цели обучения
    • Определить производную функцию заданной функции.
    • График производной функции по графику заданной функции.
    • Укажите связь между производными и непрерывностью.
    • Опишите три условия, при которых функция не имеет производной.
    • Объясните значение производной высшего порядка.

    Как мы видели, производная функции в данной точке дает нам скорость изменения или наклон касательной к функции в этой точке. Если мы продифференцируем функцию положения в данный момент времени, мы получим скорость в этот момент. Кажется разумным заключить, что знание производной функции в каждой точке даст ценную информацию о поведении функции. Однако процесс нахождения производной даже при нескольких значениях с использованием методов из предыдущего раздела быстро стал бы довольно утомительным. В этом разделе мы определяем производную функцию и изучаем процесс ее нахождения.

    Производные функции

    Функция производной дает производную функции в каждой точке области определения исходной функции, для которой определена производная. Мы можем формально определить производную функцию следующим образом.

    Определение: производная функция

    Пусть \(f\) – функция. Производная функция , обозначаемая \(f’\), является функцией, область определения которой состоит из таких значений \(x\), что существует следующий предел:

    \[f'(x)=\lim_{h→0}\frac{f(x+h)−f(x)}{h}. \label{derdef} \]

    Функция \(f(x)\) называется дифференцируемой в \(a\), если \(f'(a)\) существует. В более общем смысле функция называется дифференцируемой на \(S\), если она дифференцируема в каждой точке открытого множества \(S\), а дифференцируемой функцией является функция, в которой \(f'( x)\) существует в своей области определения.

    В следующих нескольких примерах мы используем уравнение \ref{derdef} для нахождения производной функции.

    Пример \(\PageIndex{1}\): нахождение производной функции квадратного корня

    Найдите производную \(f(x)=\sqrt{x}\).

    Решение

    Начните непосредственно с определения функции производной.

    Замените \(f(x+h)=\sqrt{x+h}\) и \(f(x)=\sqrt{x}\) на \(f'(x)= \displaystyle \lim_{ h→0}\frac{f(x+h)−f(x)}{h}\).

    \(f'(x)=\displaystyle \lim_{h→0}\frac{\sqrt{x+h}−\sqrt{x}}{h}\)  
    \(=\displaystyle\lim_{h→0}\frac{\sqrt{x+h}−\sqrt{x}}{h}⋅\frac{\sqrt{x+h}+\sqrt{ х}}{\sqrt{х+ч}+\sqrt{х}}\) Умножить числитель и знаменатель на \(\sqrt{x+h}+\sqrt{x}\) без распределения в знаменателе.
    \(=\displaystyle\lim_{h→0}\frac{h}{h\left(\sqrt{x+h}+\sqrt{x}\right)}\) Умножьте числители и упростите.
    \(=\displaystyle\lim_{h→0}\frac{1}{\left(\sqrt{x+h}+\sqrt{x}\right)}\) 92}{ч}\) Упростить
    \(=\displaystyle\lim_{h→0}\frac{h(2x−2+h)}{h}\) Вынести \(h\) из числителя
    \(=\displaystyle\lim_{h→0}(2x−2+h)\) Отменить общий множитель \(h\)
    \(=2x−2\) Оценить предел
    Упражнение \(\PageIndex{1}\)

    Найдите производную \(f(x)=x^2\). 92−2x\справа)=2x−2\). Таким образом, для функции \(y=f(x)\) каждое из следующих обозначений представляет собой производную от \(f(x)\):

    \(f'(x), \quad \dfrac{dy }{dx}, \quad y′,\quad \dfrac{d}{dx}\big(f(x)\big)\).

    Вместо \(f'(a)\) мы также можем использовать \(\dfrac{dy}{dx}\Big|_{x=a}\). Использование нотации \(\dfrac{dy}{dx}\) (называемой нотацией Лейбница) довольно распространено в технике и физике. Чтобы лучше понять эти обозначения, вспомним, что производная функции в точке — это предел наклона секущих по мере приближения секущих к касательной. Наклоны этих секущих часто выражаются в виде \(\dfrac{Δy}{Δx}\), где \(Δy\) – разность значений \(y\), соответствующая разнице в \(x \) значения, которые выражаются как \(Δx\) (рисунок \(\PageIndex{1}\)). Таким образом, производная, которую можно рассматривать как мгновенную скорость изменения \(у\) по отношению к \(х\), выражается как

    \(\displaystyle \frac{dy}{dx}= \lim_{Δx→0}\frac{Δy}{Δx}\).

    Рисунок \(\PageIndex{1}\): производная выражается как \(\dfrac{dy}{dx}=\displaystyle\lim_{Δx→0}\frac{Δy}{Δx}\).

    Построение графика производной

    Мы уже обсуждали, как построить график функции, поэтому, имея уравнение функции или уравнение функции производной, мы можем построить ее график. Учитывая оба, мы ожидаем увидеть соответствие между графиками этих двух функций, поскольку \(f'(x)\) дает скорость изменения функции \(f(x)\) (или наклон касательной строка к \(f(x)\)).

    В примере \(\PageIndex{1}\) мы обнаружили, что для \(f(x)=\sqrt{x}\), \(f'(x)=\frac{1}{2\sqrt {Икс}}\). Если мы изобразим эти функции на тех же осях, как на рисунке \(\PageIndex{2}\), мы сможем использовать графики, чтобы понять связь между этими двумя функциями. Во-первых, мы замечаем, что \(f(x)\) возрастает по всей своей области, а это означает, что наклоны ее касательных во всех точках положительны. Следовательно, мы ожидаем \(f'(x)>0\) для всех значений x в его области определения. Кроме того, по мере увеличения \(x\) наклоны касательных линий к \(f(x)\) уменьшаются, и мы ожидаем увидеть соответствующее уменьшение \(f'(x)\). +}f'(x)=+∞\), что соответствует вертикальной касательной к \(f( х)\) в \(0\). 92−2x,\; f'(x)=2x−2\). Графики этих функций показаны на рисунке \(\PageIndex{3}\). Обратите внимание, что \(f(x)\) убывает при \(x<1\). Для этих же значений \(x\), \(f'(x)<0\). Для значений \(x>1\) \(f(x)\) возрастает и \(f'(x)>0\). Кроме того, \(f(x)\) имеет горизонтальную касательную в точках \(x=1\) и \(f'(1)=0\).

    Рисунок \(\PageIndex{3}\): производная \(f'(x)<0\), где функция \(f(x)\) убывает и \(f'(x)>0\) где \(f(x)\) возрастает. Производная равна нулю там, где функция имеет горизонтальный тангенс
    Пример \(\PageIndex{3}\): набросок производной с помощью функции

    Используйте следующий график \(f(x)\) для построения графика \(f'(x)\).

    Решение

    Решение показано на следующем графике. Заметим, что \(f(x)\) возрастает и \(f'(x)>0\) на \((–2,3)\). Кроме того, \(f(x)\) убывает и \(f'(x)<0\) на \((−∞,−2)\) и на \((3,+∞)\). Также обратите внимание, что \(f(x)\) имеет горизонтальные касательные в точках \(-2\) и \(3\), а \(f'(-2)=0\) и \(f'(3)= 0\). 92−4\). На каком интервале находится график \(f'(x)\) над осью \(x\)?

    Подсказка

    График \(f'(x)\) положителен, где \(f(x)\) возрастает.

    Ответить

    \((0,+∞)\)

    Производные и непрерывность

    Теперь, когда мы можем изобразить производную, давайте рассмотрим поведение графиков. Сначала рассмотрим связь между дифференцируемостью и непрерывностью. Мы увидим, что если функция дифференцируема в точке, она должна быть там непрерывной; однако функция, непрерывная в точке, не обязательно должна быть дифференцируемой в этой точке. На самом деле функция может быть непрерывной в точке и не быть дифференцируемой в этой точке по одной из нескольких причин.

    Дифференцируемость подразумевает непрерывность

    Пусть \(f(x)\) – функция и \(a\) находится в ее области определения. Если \(f(x)\) дифференцируема в \(а\), то \(f\) непрерывна в \(а\).

    Доказательство

    Если \(f(x)\) дифференцируема в \(a\), то \(f'(a)\) существует и, если положить \(h = x – a\), мы имеют \( x = a + h \), и поскольку \(h=x-a\to 0\), мы можем видеть, что \(x\to a\).

    Тогда

    \[ f'(a) = \lim_{h\to 0}\frac{f(a+h)-f(a)}{h}\nonumber \]

    можно переписать как

    \(f'(a)=\displaystyle \lim_{x→a}\frac{f(x)−f(a)}{x−a}\).

    Мы хотим показать, что \(f(x)\) непрерывно в \(a\), показав, что \(\displaystyle \lim_{x→a}f(x)=f(a).\) Таким образом ,

    \(\begin{align*} \displaystyle \lim_{x→a}f(x) &=\lim_{x→a}\;\big(f(x)−f(a)+f( a)\big)\\[4pt]
    &=\lim_{x→a}\left(\frac{f(x)−f(a)}{x−a}⋅(x−a)+f( a)\right) & & \text{Умножить и разделить}(f(x)−f(a))\text{ на }x−a.\\[4pt]
    &=\left(\lim_{x→ a}\frac{f(x)−f(a)}{x−a}\right)⋅\left( \lim_{x→a}\;(x−a)\right)+\lim_{x→ а}ж(а)\\[4pt]
    &=f'(a)⋅0+f(a)\\[4pt]
    &=f(a). \end{align*}\)

    Следовательно, поскольку \(f(a)\) определено и \(\displaystyle \lim_{x→a}f(x)=f(a)\), мы заключаем, что \(f\) непрерывна в \(a\).

    Мы только что доказали, что дифференцируемость влечет непрерывность, но теперь мы рассмотрим, влечет ли непрерывность дифференцируемость. Чтобы определить ответ на этот вопрос, мы исследуем функцию \(f(x)=|x|\). Эта функция всюду непрерывна; однако \(f'(0)\) не определено. Это наблюдение приводит нас к мысли, что непрерывность не влечет дифференцируемости. Давайте исследовать дальше. Для \(f(x)=|x|\), 92}}=+∞\).

    Таким образом, \(f'(0)\) не существует. Беглый взгляд на график \(f(x)=\sqrt[3]{x}\) проясняет ситуацию. Функция имеет вертикальную касательную в точке \(0\) (рисунок \(\PageIndex{5}\)).

    Рисунок \(\PageIndex{5}\): функция \(f(x)=\sqrt[3]{x}\) имеет вертикальную касательную в точке \(x=0\). Он непрерывен в точке \(0\), но не дифференцируем в точке \(0\).

    Функция \(f(x)=\begin{cases} x\sin\left(\frac{1}{x}\right), & & \text{ if } x≠0\\0, & & \ text{ if } x=0\end{cases}\) также имеет производную, которая демонстрирует интересное поведение при \(0\).

    Мы видим, что

    \(f'(0)=\displaystyle \lim_{x→0}\frac{x\sin\left(1/x\right)−0}{x−0}= \lim_ {x→0}\sin\left(\frac{1}{x}\right)\).

    Этого предела не существует, в основном потому, что наклон секущих постоянно меняет направление по мере приближения к нулю (рис. \(\PageIndex{6}\)).

    Рисунок \(\PageIndex{6}\): функция \(f(x)=\begin{cases} x\sin\left(\frac{1}{x}\right), & & \text{, если } x≠0\\0, & & \text{ если } x=0\end{cases}\) не дифференцируемо в \(0\).

    Итого:

    1. Заметим, что если функция не непрерывна, она не может быть дифференцируемой, поскольку каждая дифференцируемая функция должна быть непрерывной. Однако, если функция непрерывна, она может не быть дифференцируемой.
    2. Мы видели, что \(f(x)=|x|\) не может быть дифференцируемым в \(0\), потому что предел наклона касательных линий слева и справа не одинаков. Визуально это привело к острому углу на графике функции в точке \(0.\). Отсюда заключаем, что для того, чтобы быть дифференцируемой в точке, функция должна быть в этой точке «гладкой».
    3. Как мы видели на примере \(f(x)=\sqrt[3]{x}\), функция не может быть дифференцируемой в точке, где есть вертикальная касательная.
    4. Как мы видели с \(f(x)=\begin{cases}x\sin\left(\frac{1}{x}\right), & & \text{ if } x≠0\\0, & &\text{ if } x=0\end{cases}\) функция может не быть дифференцируемой в точке и более сложными способами.
    Пример \(\PageIndex{4}\): кусочная функция, которая является непрерывной и дифференцируемой 92+bx+c, & & \text{, если }x
    <−10\\−\frac{1}{4}x+\frac{5}{2}, & & \text{, если } x≥−10\ end{cases}\), где \(x\) и \(f(x)\) указаны в дюймах. Для плавного движения автомобиля по трассе функция \(f(x)\) должна быть одновременно непрерывной и дифференцируемой в точке \(−10\). Найдите значения \(b\) и \(c\), которые делают \(f(x)\) одновременно непрерывным и дифференцируемым.

    Рисунок \(\PageIndex{7}\): Чтобы автомобиль двигался плавно по трассе, функция должна быть одновременно непрерывной и дифференцируемой.

    92−10b+c=10−10b+c\)

    и \(f(−10)=5\), мы должны иметь \(10−10b+c=5\). 2+bx+c−5}{x+10}\\[4pt] 92, & & \text{ если } x≥3\end{cases}\) как непрерывны, так и дифференцируемы в \(3\).

    Подсказка

    Используйте пример \(\PageIndex{4}\) в качестве руководства.

    Ответить

    \(a=6\) и \(b=−9\)

    Производные высшего порядка

    Производная функции сама по себе является функцией, поэтому мы можем найти производную производной. Например, производная функции положения — это скорость изменения положения или скорость. Производная скорости — это скорость изменения скорости, то есть ускорение. Новая функция, полученная дифференцированием производной, называется второй производной. Кроме того, мы можем продолжать брать производные, чтобы получить третью производную, четвертую производную и так далее. В совокупности они обозначаются как 92−3h}{ч}\) Упростите числитель. \(=\displaystyle \lim_{h→0}(4x+2h−3)\) Вынесите \(h\) в числителе и сократите с \(h\) в знаменателе. \(=4x−3\) Возьмите предел.

    Затем найдите \(f”(x)\), взяв производную от \(f'(x)=4x−3.\)

    \), найти \(a(t).\)

    Подсказка

    Используйте пример \(\PageIndex{6}\) в качестве руководства.

    Ответить

    \(а(т)=6т\)

    Основные понятия

    • Производной функции \(f(x)\) называется функция, значение которой в точке \(x\) равно \(f'(x)\).
    • График производной функции \(f(x)\) связан с графиком функции \(f(x)\). Где \(f(x)\) имеет касательную с положительным наклоном, \(f'(x)>0\). Где \(f(x)\) имеет касательную с отрицательным наклоном, \(f'(x)<0\). Где \(f(x)\) имеет горизонтальную касательную, \(f'(x)=0.\) 9{\text{th}}\) производная.

    Ключевые уравнения

    • Производная функция

    \(f'(x)=\displaystyle \lim_{h→0}\frac{f(x+h)−f(x)}{h}\)

    Глоссарий

    производная функция
    дает производную функции в каждой точке области определения исходной функции, для которой определена производная
    дифференцируемый по \(а\)
    функция, для которой \(f'(a)\) существует, дифференцируема в точке \(a\) 9{\text{th}}\) производная, называется производной высшего порядка

    Авторы и авторство


    Эта страница под названием 3. 2: Производная как функция распространяется под лицензией CC BY-NC-SA 4.0 и была создана, изменена и/или курирована Гилбертом Стрэнгом и Эдвином «Джедом» Херманом (OpenStax) через исходный контент, который был отредактирован в соответствии со стилем и стандартами платформы LibreTexts; подробная история редактирования доступна по запросу.

    1. Вернуться к началу
      • Была ли эта статья полезной?
      1. Тип изделия
        Раздел или Страница
        Автор
        ОпенСтакс
        Лицензия
        CC BY-NC-SA
        Версия лицензии
        4,0
        Программа OER или Publisher
        ОпенСтакс
        Показать страницу TOC
        нет
      2. Теги
        1. автор @ Эдвин «Джед» Герман
        2. автор@Гилберт Странг
        3. производная функция
        4. Дифференцируемая функция
        5. дифференцируемый на S
        6. производная высшего порядка
        7. источник@https://openstax. org/details/books/calculus-volume-1

      Производные функции: примеры и формулы

      Производные, возможно, являются одним из самых важных понятий, которые мы можем изучать в математике. Почему ты спрашиваешь? Потому что они необходимы во многих приложениях! Вот некоторые из них:

      • Производные помогают нам определить взаимосвязь между положением, скоростью и ускорением объекта в физике.

      • Они могут сообщить нам скорость изменения таких вещей, как температура, прибыль и численность населения в определенный момент времени.

      • Производные являются частью моделей оптимизации для улучшения принятия решений в таких отраслях, как здравоохранение, экономика, бизнес, наука, инженерия и т. д. .

      Как мы узнали из нашей статьи о производных, существует метод нахождения производной функции исходной функции. Это означает, что мы можем определить функцию, которая дает нам производную исходной функции в каждой точке области определения исходной функции.

      • Производная формулы функции
      • Вычисление производной функции
      • Обозначения производных
      • Производная тригонометрической и обратной тригонометрической функций – примеры
      • Производная экспоненциальной и логарифмической функций – примеры

      Производная функции Формула

      Производная функция дает нам производную функции в каждой точке области определения функции, в которой производная определена.

      • Это означает отсутствие вертикальных касательных, разрыва прыжка, разрыва устранимого разрыва и острых точек.

      • Другими словами, предел в приведенном ниже определении должен существовать.

      Допустим, у нас есть функция, обозначенная \(f\). Его производная функция , обозначаемая \(f’\), представляет собой функцию, область определения которой состоит из значений \(x\), таких, что предел ниже существует:

      \[f'(x) = \lim_{ h \to 0} \frac{f(x+h)-f(x)}{h}\]

      Это называется предельным определением производной , а иногда просто определением производной .

      Нахождение производной функции с использованием этого предела иногда называют доказательством производной по первому принципу.

      • Функция \(f(x)\) считается дифференцируемой в точке \(a\), если ее производная в этой точке \(f'(a)\) существует.
      • Итак, в общем случае функция считается дифференцируемой на открытом множестве \(S\), если она дифференцируема в каждой точке этого множества.
        • Дифференцируемая функция — это функция, в которой \(f'(x)\) существует в своей области определения.

      Вычисление производной функции

      В следующих примерах мы используем определение производной функции , чтобы найти производную функции.

      Найдите производную функции квадратного корня:

      \[f(x) = \sqrt{x}\]

      Решение :

      1. Подставьте \(f(x+h) = \sqrt {x+h}\) и \(f(x) = \sqrt{x}\) в \(f'(x) = \lim_{h \to 0} \frac{f(x+h)-f (х)}{ч}\).

      \[f'(x) = \lim_{h \to 0} \frac{\sqrt{x+h} – \sqrt{x}}{h}\]

      2. Умножить числитель и знаменатель на \(\sqrt{x+h} + \sqrt{x}\).\[f'(x) = \lim_{h \to 0} \frac{\sqrt{x+h} – \sqrt{x} }{h} \cdot \frac{\sqrt{x+h} + \sqrt{x}}{\sqrt{x+h} + \sqrt{x}}\]3. Умножьте числители и упростите, не распределяя знаменатель.\[f'(x) = \lim_{h \to 0} \frac{h}{h\left(\sqrt{x+h} + \sqrt{x} \справа)}\]4. Отмените \(h\).\[f'(x) = \lim_{h \to 0} \frac{1}{\sqrt{x+h} + \sqrt{x}}\]5. Установите \(h = 0\) и оцените предел.\[ \bf{ f'(x) = \frac{1}{2\sqrt{x}} }\] 9{2}}{h}\]

      4. Вынесите \(h\) из числителя.

      \[f'(x) = \lim_{h \to 0} \frac{h(2x + h)}{h}\]

      5. Отменить \(h\).

      \[f'(x) = \lim_{h \to 0} (2x + h)\]

      6. Установить \(h = 0\) и вычислить предел.

      \[ \bf{ f'(x) = 2x } \]

      Обозначения производных

      Существует большое разнообразие обозначений для выражения производной функции. В приведенном выше примере мы доказали, что функция:

      \[f(x) = x^{2}\] 9{2}\right) = 2x\]

      Подводя итог, можно сказать, что для любой функции \(y = f(x)\) каждое из следующих обозначений представляет собой производную от \(f(x)\):

      \[f'(x), \frac{d}{dx} (f(x)), y’, \frac{dy}{dx} \]

      При взятии производной в точке можно также заменить обозначение \(f'(a)\) с обозначением:

      \[ \left. \frac{dy}{dx} \right|_{x=a} \]

      Чтобы лучше понять обозначения Лейбница, \( \frac{dy}{dx} \), мы должны помнить, что производная функции в точке:

      Обычно наклоны этих секущих выражают следующим образом:

      \[ \frac{\Delta y}{\Delta x} \]

      значения, а \(\Delta x \) — разница в значениях x.

      Следовательно, производная , или мгновенная скорость изменения y по отношению к x , может быть выражена как:

      \[ \frac{dy}{dx} = \lim_{\Delta x \to 0} \frac{\Delta y}{\Delta x} \]

      Графическое представление производной в виде \( \frac{dy}{dx} \) – StudySmarter Originals

      Производная триггерной и обратной триггерных функций – примеры

      Что такое производная \(sin(x)\)?

      Решение :

      1. Примените определение производной.

      \[ \frac{d}{dx} sin(x) = \lim_{h \to 0} \frac{sin(x+h) – sin(x)}{h} \]

      2. Использование сумма углов триггерная идентичность, чтобы получить:

      \[ \frac{d}{dx} sin(x) = \lim_{h \to 0} \frac{ \left[sin(x)cos(h) + cos (x)sin(h)\right] – sin(x)}{h} \]

      3. Переставьте члены с \(sin(x)\) рядом друг с другом.

      \[ \frac{d}{dx} sin(x) = \lim_{h \to 0} \frac{cos(x)sin(h) – sin(x) + sin(x)cos(h) }{h} \]

      4. Вынесите за скобки \(sin(x)\).

      \[ \frac{d}{dx} sin(x) = \lim_{h \to 0} \frac{cos(x)sin(h) – sin(x)(1 – cos(h))} {h} \]

      5. Примените следующие законы пределов: правило постоянного кратного и правило разности.

      \[ \frac{d}{dx} sin(x) = cos(x) \left(\lim_{h \to 0} \frac{sin(h)}{h} \right) – sin(x ) \left( \lim_{h \to 0} \frac{1 – cos(h)}{h} \right) \]

      6. Если построить график \( \frac{sin(h)}{h} \), мы увидим, что предел при \(h \to 0\) равен \(1\).

      Это также можно доказать с помощью теоремы о сжатии.

      Предел как \(h \to 0\) для \( \frac{sin(h)}{h} \) равен \(1\) – StudySmarter Originals

      7. Если построить график \( \frac{ 1 – cos(h)}{h} \), мы видим, что предел при \(h \to 0\) равен \(0\).

      Предел как \(h \to 0\) для \( \frac{1-cos(h)}{h} \) равен \(0\) – StudySmarter Originals

      8. Итак, мы можем подключить \(1\) для первого предела и \(0\) для второго предела и упростить. 9{-1}(x) = arcsin(x) \)? 1, 2

      Прежде чем мы начнем, знайте, что процесс нахождения этой производной возможен с использованием определения производной (известного как доказательство по первому принципу), однако это сложный и длительный процесс, который также требует сложных алгебраических вычислений. манипуляция. Гораздо проще найти эту производную, используя некоторые производные процессы, которые вы, возможно, еще не знаете:

      • Неявное дифференцирование и
      • Цепное правило

      Пожалуйста, ознакомьтесь с нашими статьями по этим темам, чтобы полностью понять этот процесс. 9{-1}(x) \\ sin(y) & = x \end{align} \]

      2. Затем, используя неявное дифференцирование и цепное правило, мы берем производную обеих сторон и находим \( y ‘\).

      \[ \begin{align} \frac{dy}{dx} (sin(y)) & = \frac{dy}{dx} (x) \\ \frac{d (sin(y))}{ dy} \cdot \frac{dy}{dx} & = 1 \\ (cos(y)) \cdot y’ & = 1 \\ y’ & = \frac{1}{cos(y)} \end{ align} \]

      3. Используя тождество триггера Пифагора, мы можем переписать это уравнение в терминах \( x = sin(y) \). 9{x} \cdot ln (b) } \]

      Производные логарифмических функций – примеры

      Какова производная \(L(x) = ln(x)\)?

      Решение :

      1. Примените определение производной.

      \[ L'(x) = \lim_{h \to 0} \frac{ln(x+h)-ln(x)}{h} \]

      2. Используйте правило частных логарифмов, \ ( ln(a)-ln(b) = ln\left( \frac{a}{b} \right) \), чтобы переписать предел как:

      \[ L'(x) = \lim_{h \to 0} \frac{ ln \left( \frac{x+h}{x} \right)}{h} \] 9{\frac{1}{y}} = \frac{1}{x} ln (e) = \frac{1}{x} \]

      9. Следовательно,

      \[ \bf{ L'( x) = \frac{1}{x} } \]

      Какова производная от \( A(x) = log_{a}(x) \)?

      Мы можем найти эту производную, используя две вещи, которые мы уже знаем:

      1. Производная \( ln(x) \) равна \( \frac{1}{x} \).
      2. Изменение основного правила логарифмирования: \( log_{a}(x) = \frac{ln(x)}{ln(a)} \)

      Решение :

      1. Используйте замену базового правила, чтобы переписать функцию как:

      \[ A(x) = \frac{ln(x)}{ln(a)} \]

      2. Теперь мы можем взять производную.

      \[\begin{align}A'(x) & = \frac{d}{dx} \left( \frac{ln (x)}{ln(a)} \right) \\& = \frac {1}{ln(a)} \cdot \frac{d}{dx} (ln(x)) \\& = \frac{1}{ln(a)} \cdot \frac{1}{x} \\\bf{ A'(x) } & = \bf{ \frac{1}{x ln(a)} }\end{align}\]

      Производные функции – ключевые выводы

      • Мы можем найти производная функция функции с использованием предельного определения производной:

      \[ f'(x) = \lim_{h \to 0} \frac{f(x+h)-f(x)}{h} \]

      • Для любой функции \(y = f(x)\), каждое из следующих обозначений представляет собой производную от \(f(x)\):

      \[ f'(x), \, \frac{d}{dx}, \, f ‘(f(x)), \, y’, \, \frac{dy}{dx} \]

      • Использование предельного определения производной – утомительный процесс! Вот почему математики разработали несколько правил дифференцирования, чтобы упростить задачу. {2}(x) \] 9{2}} } \, \mbox{где,} \, x \neq 0 \]

      Ссылки

      1. https://www.mechamath.com/calculus/derivative-of-arcsin-inverse -sine-with-proof-and-graphs/
      2. https://www.mathdoubts.com/derivative-of-sin-inverse-function-proof/

      Примеры и решения – mathsathome.com

      Как использовать Видеоурок по цепному правилу

      Цепное правило с тригонометрическими функциями

      Что такое цепное правило?

      Цепное правило используется для вычисления производной сложной функции. Формула цепного правила гласит, что dy / dx = dy / du × du / dx 5 9. Другими словами, дифференцируйте внешнюю функцию, сохраняя внутреннюю функцию неизменной, а затем умножайте ее на производную внутренней функции.

      Цепное правило: нотация Лейбница

      Цепное правило: обозначение функций

      Цепное правило используется, когда функция находится внутри другой функции.

      Цепное правило определяется как, где u является функцией 𝑥 ( u = g(x) ), а y является функцией u ( y = f(u) ).

      В качестве альтернативы цепное правило можно записать в функциональном обозначении как F'(𝑥) = f'(g(𝑥)).g'(𝑥), где F(𝑥) = f(g(𝑥)). g(𝑥) — внутренняя функция, а f(𝑥) — внешняя функция.

      Другими словами, цепное правило требует найти производную внешней функции, сохраняя внутреннюю функцию неизменной, а затем умножив ее на производную внутренней функции.

      Чтобы использовать цепное правило, необходимы следующие правила:

      • Функция должна быть составной функцией двух или более функций
      • Такие функции должны быть дифференцируемы сами по себе

      Как сделать цепное правило

      выполнить цепное правило:

      1. Различать внешнюю функцию, сохраняя внутреннюю функцию неизменной.
      2. Умножьте это на производную от внутренней функции.

      Например, дифференцировать (4𝑥 – 3)

      5 с использованием цепного правила

      В этом примере мы будем использовать цепное правило шаг за шагом. Ниже мы будем использовать метод формулы цепного правила.

      Мы определяем 4𝑥 – 3 как внутреннюю функцию, а ( ) 5 как внешнюю функцию.

      Шаг 1. Дифференцируйте внешнюю функцию, оставив внутреннюю функцию неизменной

      Внешняя функция — это ( ) 5 . Мы различаем это, как если бы 𝑥 5 .

      𝑥 5 дифференцируется в 5𝑥 4 , поэтому мы пишем ( ) 5 , дифференцированный как 5( ) 4 .

      Мы сохраняем внутреннюю функцию 4𝑥-3 прежней, поэтому пишем 5(4𝑥 – 3) 4 .

      Однако, поскольку у нас в скобках 4𝑥 – 3, а не только 𝑥, мы должны также включить шаг 2.

      Шаг 2. Умножить это на производную от внутренней функции

      Из шага 1 у нас уже есть 5 (4𝑥 – 3) 4 и теперь мы должны умножить это на производную от внутренней функции.

      Внутренняя функция — это функция, заключенная в скобки. Это 4𝑥 – 3.

      . Мы дифференцируем 4𝑥 – 3, чтобы получить 4. Таким образом, мы должны умножить результат 5(4𝑥 – 3) 4 на 4.

      . Мы приходим к ответу f'(𝑥) = 20(4𝑥 – 3) 4 .

      Теперь мы будем использовать формулу цепного правила, чтобы дифференцировать эту функцию. Ответ будет таким же.

      Формула цепного правила утверждает, что F'(𝑥) = f'(g(𝑥). g'(𝑥), где g(𝑥) — внутренняя функция, а f(𝑥) — внешняя функция.

      Следовательно, для F(𝑥) = (4𝑥 – 3) 5 , g(𝑥) = 4𝑥 – 3 и f(𝑥) = 𝑥 5 .

      f'(𝑥) равно 5𝑥 4 .

      f'(g(𝑥)) означает замену g(𝑥) вместо 𝑥 в f'(𝑥).

      Следовательно, f'(g(𝑥)) = 5(4𝑥 – 3) 4 .

      g'(𝑥) = 4.

      Следовательно, F'(𝑥) = f'(g(𝑥).g'(𝑥) становится F'(𝑥) = 5(4𝑥 – 3) 4 × 4

      Это упрощается до F'(𝑥) = 20(4𝑥 – 3) 4 .

      Цепное правило с тригонометрией

      Цепное правило используется для дифференциации тригонометрических функций, содержащих другую функцию. Дифференцируйте тригонометрическую функцию, оставив внутреннюю функцию неизменной, а затем умножьте ее на производную от внутренней функции.

      Здесь могут быть полезны следующие правила дифференцирования тригонометрических функций от sin(𝑥), cos(𝑥) и tan(𝑥):

      Например, дифференцировать cos(2𝑥) с помощью цепного правила ) можно записать как внутреннюю функцию 2𝑥 и внешнюю функцию cos().

      Шаг 1. Дифференцируем внешнюю функцию, сохраняя внутреннюю функцию той же

      cos(𝑥) дифференцируется в -sin(𝑥) и поэтому, сохраняя внутреннюю функцию как 2𝑥, а не 𝑥, мы получаем -sin(2𝑥 ).

      Шаг 2. Умножьте это на производную внутренней функции

      Внутренняя функция равна 2𝑥, а ее производная равна 2.

      Мы умножаем -sin(2𝑥) на 2, чтобы получить f'(x) = -2sin (2𝑥).

      Цепное правило можно применять к тригонометрическим функциям, возведенным в степень.

      1. Запишите тригонометрическую функцию как внутреннюю функцию в скобках и степень как внешнюю функцию.
      2. Уменьшите степень и вычтите единицу из степени, сохранив тригонометрическую функцию внутри прежней.
      3. Умножьте это на производную тригонометрической функции.

      Например, продифференцируйте sin

      4 (2𝑥), используя цепное правило

      Шаг 1. Сначала запишите sin 4 (2𝑥) как [sin(2𝑥)] 4 .

      Шаг 2. Уменьшите степень и вычтите единицу из степени, сохранив тригонометрическую функцию той же.

      Мы опускаем 4 перед скобками, степень становится равной 3, и мы сохраняем sin(2𝑥) внутри скобок, что дает нам 4[ sin(2𝑥) ] 3 .

      Шаг 3. Умножить на производную sin(2𝑥). Здесь мы используем цепное правило, поэтому sin(𝑥) — это внешняя функция, которая дифференцируется в cos(𝑥).

      Мы оставляем 2𝑥 внутри, чтобы получить cos(2𝑥). Затем мы умножаем это на производную от 2𝑥, которая равна 2,9.0046

      дифференцированный sin(2𝑥) равен 2cos(2𝑥).

      Итак, мы умножаем 4[sin(2𝑥)] 3 на 2cos(2𝑥).

      f'(𝑥) = 8sin 3 (2𝑥)cos(2𝑥)

      Цепное правило с логарифмами

      Цепное правило утверждает, что для y = ln(u), dy =

      1225 1 / u × du / d𝑥 . Другими словами, дифференцируйте внутреннюю функцию, а затем разделите ее на внутреннюю функцию. Например, если y = ln(𝑥 2 + 3 𝑥 ), dy / d𝑥 = (2𝑥 + 3)/( 𝑥 2 + 3 𝑥 ).

      Производная от y = ln(u) равна 1 / u × du / d𝑥 .

      u — это функция внутри функции ln. Для y = ln(𝑥 2 + 3𝑥), u = 𝑥 2 + 3𝑥.

      Следовательно, du / d𝑥 = 2𝑥 + 3.

      Производная dy / d𝑥 =.

      Мы можем записать это как .

      Цепное правило позволяет различать функции журнала. Производная y = ln ( f(𝑥) ) может быть записана как:

      или .

      Чтобы дифференцировать log любого основания, если y = log a f(𝑥), то .

      Например, продифференцируем y = log 5 (2𝑥 + 1):

      Здесь f(𝑥) = 2𝑥 + 1 и, следовательно, f'(𝑥) = 2.

      Подставляя их в формулу, получаем :

      Цепное правило с экспоненциальными функциями

      Производная y = E 𝑥 IS DY / D𝑥 = E 𝑥 и с использованием цепного правила, . Производное Y = E F (44344 4101534.9.700 .700 .700 .700 .7.700 .00 343434343434343434 34343434343434343 (344.9.9.700). dy / d𝑥 = f'(𝑥 ). e f ( 𝑥 ) . Просто продифференцируйте мощность e и умножьте ее на исходную функцию.

      Например, дифференцируйте e 5𝑥+3 .

      В этом примере f(𝑥) = 5𝑥 + 3 и, следовательно, f'(𝑥) = 5.

      Чтобы найти производную от y = e 5𝑥+3 , мы просто умножаем ее на 5.

      Получаем dy / d𝑥 = 5 e 5𝑥+3 .

      Применение цепного правила с тремя функциями

      Цепное правило можно применять к композиции из трех функций. Если y(𝑥) = h(g(f(x))), то y'(𝑥) = f'( 𝑥 ) . g'(f( 𝑥 )) . h'(g(f( 𝑥))). Однако проще применить цепное правило дважды, чтобы различать три функции.

      Например, дифференцировать f(𝑥) = sin 2 (5𝑥).

      Здесь у нас есть композиция из 3-х функций.

      1. Самая внутренняя функция — 5𝑥.
      2. Действие на 5𝑥 есть функция sin
      3. Наконец, функция sin возводится в степень 2

      Запишем sin 2 (5𝑥) как [sin(5𝑥)] 2 .

      Мы дифференцируем, используя цепное правило, так что мы дифференцируем внешнюю функцию, сохраняя внутреннюю функцию неизменной, а затем умножаем на производную внутренней функции.

      Дифференцируя внешнюю квадратичную функцию, сохраняя внутреннюю функцию неизменной, мы получаем 2[ sin(5𝑥) ]. Сила 2 уменьшилась, мы вычли 1 из мощности и оставили грех (5𝑥) внутри прежним.

      Теперь нам нужно умножить это значение на производную внутренней функции sin(5𝑥). Это требует повторного использования цепного правила.

      Мы дифференцируем внешнюю функцию sin, чтобы получить cos, и сохраняем внутреннюю функцию 5𝑥 такой же. Получаем cos(5𝑥). Нам нужно умножить это на производную внутренней функции 5𝑥. Мы находим, что производная от sin(5𝑥) равна 5cos(5𝑥).

      Теперь мы умножаем предыдущий результат 2[ sin(5𝑥) ] на 5cos(5𝑥), чтобы получить окончательную производную 10sin(5𝑥)cos(5𝑥).

      Примеры цепного правила с решениями

      Вот несколько примеров использования цепного правила для различения различных функций:

      \(f”(x)= \displaystyle \lim_{h→0}\frac{f'(x+h)−f'(x)}{h}\) Используйте \(f'(x)=\displaystyle \lim_{h→0}\frac{f(x+h)−f(x)}{h}\) с \(f ‘(x)\) в место \(f(x).\)
      \(=\displaystyle \lim_{h→0}\frac{(4(x+h)−3)−(4x−3)}{h}\) Замените \(f'(x+h)=4(x+h)−3\) и \(f'(x)=4x−3.\)
      \(=\displaystyle \lim_{h→0}4\) Упростить.
      \(=4\) Возьмите предел.
      Функция Расчет Произвольная = 15 (5𝑥 + 1) 2
      F (𝑥) = sin (5𝑥) cos (5𝑥) × 5 f ‘(𝑥) = 5cos (5𝑥)
      (𝑥) = 5cos (5𝑥)
      (𝑥) = ) = cos(10𝑥) -sin(10𝑥) × 10 f'(𝑥) = -10sin(10𝑥)
      f(𝑥) = -3tan(2𝑥) -3SEC 2 (2𝑥) × 2 F ‘(𝑥) = -6 сек 2 (2𝑥)
      F (𝑥) = 10E 4𝑥 10 10 10.7 409087 9087 10. 107

    2. 7
    3. 7
    4. 10. 1009037 409037. F ‘(𝑥) = 40e 4𝑥
      F (𝑥) = LN (𝑥 3 ) 1 /(𝑥 3 ) × 3𝑥 20000000009 темнонный )/(𝑥 3 ) f'(𝑥) = 3 / 𝑥

      Когда использовать Цепное правило

      Цепное правило используется для дифференцирования любой сложной функции вида y = f(g(𝑥)). Это функция, которая имеет внутреннюю функцию с внешней функцией, примененной к ней. Например, y = (3 𝑥 + 2) 5 состоит из функций g( 𝑥) = 3 𝑥 + 2 и f( 𝑥 ) = 5 9002 90

      Вот несколько примеров того, когда и когда не следует использовать цепное правило:

      Пример Должен ли я использовать цепное правило? Почему?
      y = (3𝑥+2) 5 Да Внутренняя функция 3𝑥+2, а внешняя функция 𝑥 5 .
      y = 𝑥 5 Нет Есть только одна функция 𝑥 5 . Его можно дифференцировать напрямую.
      y = (𝑥+3) sin(𝑥) Нет Это произведение двух функций. Используйте правило произведения.
      y = sin(2𝑥+1) Да Внутренняя функция 2𝑥+1, а внешняя функция sin(𝑥)
      y = (𝑥+5)

      / 5)
      Это дробь с функцией числителя и знаменателя. Используйте правило отношения.
      y = ln(𝑥 4 -𝑥) Да Внутренняя функция 𝑥 4 -𝑥. Внешняя функция — это ln(𝑥).
      у = е 2𝑥 Да Внутренняя функция — 2𝑥, а внешняя функция — e 𝑥 .

      Chain Rule Proof

      To prove the chain rule, consider dy / dx as a limit of Δ y / Δ𝑥 as 𝑥 стремится к нулю. Это можно записать в виде пределов Δ y / Δu × Δ u / Δ 𝑥 Evaluating these limits as 𝑥 and u tend to zero, we derive the chain rule as dy / d 𝑥 = dy / du × dy / d 𝑥 .

      Чтобы доказать цепное правило:

      1. Запишите dy / d𝑥 как предел Δy / Δ9x как .
      2. Запишите предел Δy / Δ𝑥 как Δy / Δu × Δu / Δ𝑥 . Δu — это просто число, так что это можно сделать.
      3. Разделить предел на 2 предела: Δy / Δu и Δu / Δ𝑥 .
      4. Поскольку Δ𝑥 стремится к 0, то же самое происходит и с Δu.
      5. Оцените оба предела, чтобы получить dy / d𝑥 = dy / du × du / d𝑥 .

      Производные составных функций – GeeksforGeeks

      Производные являются неотъемлемой частью исчисления. Они помогают нам вычислять скорость изменения, максимумы и минимумы функций. Производные по определению даются с использованием пределов, которые называются первой формой производной. Мы уже знаем, как вычислять производные стандартных функций, но иногда нам приходится иметь дело со сложными математическими функциями, состоящими из более чем двух функций. Становится трудно и громоздко вычислять производную для таких функций методом грубой силы. Становится важным узнать о правилах и методах, облегчающих наши расчеты. Одним из них является цепное правило, позволяющее вычислять производные сложных функций. Давайте посмотрим на это правило формально.

      Составные функции и цепное правило 

      Допустим, у нас есть функция f(x) = (x + 1) 2 , для которой мы хотим вычислить производную. Такие функции называются составными функциями, что означает, что они состоят из более чем одной функции. Обычно они имеют вид g(x) = h(f(x)) или также могут быть записаны как g = hof(x). В нашем случае заданная функция f(x) = (x + 1) 2 состоит из двух функций,

      f(x) = g(h(x)), где g(x) = x 2 и h(x) = x + 1.

      Например,

      f(x) = (x + 1) 2

      ⇒ f(x) = x 2 + 1 + 2x

      Дифференцирование функции по х, расширяйте его каждый раз, а затем различайте его. В этих случаях цепное правило становится необходимым.

      Цепное правило

      Пусть f — функция с действительным знаком, представляющая собой комбинацию двух функций, «u» и «v», то есть f = v o u. Допустим, t = u(x) и если для обеих функций «u» и «v» существуют и .

      Цепное правило можно распространить на любое количество составных функций. Например,

      f = (w o u) o v. Если t = v(x) и s = u(t), то

      Допустим, у нас есть функция f(x) = sin(x 2

      Эта функция представляет собой составную функцию, состоящую из двух функций. Если t = u(x) = x 2 и v(t) = sin(t), то

      f(x) = (v o u)(x) = v(u(x)) = v(x 2 ) = sin x 2

      Положим t = u(t) = x 2 . и существует. Следовательно, по цепному правилу 

      Метод, альтернативный цепному правилу (краткий прием для поиска производной с помощью цепного правила)

      Цепное правило также можно применять с помощью сокращенного метода. Это поясняется на примере, допустим, у нас есть функция f(x) = (sin(x)) 2 . В общем, мы не используем подход композиции функций для дифференциации функций. Мы различаем «внутреннюю функцию» и «внешнюю функцию». Затем дифференцируйте внешнюю функцию, оставляя внутреннюю функцию в покое, и продолжайте в том же духе в иерархии.

      Как правило, этот сокращенный метод используется для простого вычисления производных для различных функций.

      Давайте рассмотрим некоторые проблемы с этим правилом, 

      Примеры задач 

      Задачи на производные полиномиальной функции и сложной функции с использованием цепного правила

      Вопрос 1. Найдите производную функции f(x) = (x + 2) 2 .

      Решение:

      Эта функция является составной функцией,

      Вопрос 2: Найдите деривативную для функции f (x) = (X 9090 3 60054.0902 2 + 1) 10

      РЕШЕНИЕ:

      446 3: Findiation). + 1) 5

      РЕШЕНИЕ:

      .0045 Вопрос 4: Найдите производную функции f(x) = sin(tanx + 5).

      Solution: 

       f(x) = sin(tanx + 5)

      Problem on derivatives of Power функции с использованием цепного правила

      Вопрос 5: Найдите производную функции, f(x) = e (2x + 5) .

      Решение:

      F (x) = E (2x + 5)

      ⇒ Derives

      40044. shivivis 400449004.shivivis.shivivis9004.salives9005 40044

      005. Вопрос 6: Найдите производную функции f(x) = | х + 1 |.

      Решение:

      Допустим, f(x) = |x + 1|.

      Мы знаем, что такие функции модуля |x| представляют √x 2 . Итак, каждую модульную функцию можно преобразовать таким образом, чтобы найти производную.

      f(x) =  

      ⇒ 6x 9004 | = x + 1, таким образом 

      Когда x < -1 |x + 1| = -(x + 1), таким образом 

      Когда x = -1, производная не определена.

      Вопрос 7: Найдите производную функции f(x) = | 2х – 1 |.

      Решение:

      Допустим, f(x) = |2x – 1|.

      f(x) =  

      Когда x >  |2x – 1| = 2x – 1, таким образом 

      Когда x <  |2x – 1| = -(2x – 1), таким образом 

      Когда x = , производная не определена. 4\). 9{х у г}\).

       >>> выражение = выражение (x*y*z)
      >>> diff(expr, x, y, y, z, z, z, z)
       3 2 ⎛ 3 3 3 2 2 2 ⎞ x⋅y⋅z
      х ⋅y ⋅⎝x ⋅y ⋅z + 14⋅x ⋅y ⋅z + 52⋅x⋅y⋅z + 48⎠⋅ℯ
      >>> diff(expr, x, y, 2, z, 4)
       3 2 ⎛ 3 3 3 2 2 2 ⎞ x⋅y⋅z
      х ⋅y ⋅⎝x ⋅y ⋅z + 14⋅x ⋅y ⋅z + 52⋅x⋅y⋅z + 48⎠⋅ℯ
      >>> diff(expr, x, y, y, z, 4)
       3 2 ⎛ 3 3 3 2 2 2 ⎞ x⋅y⋅z
      х ⋅y ⋅⎝x ⋅y ⋅z + 14⋅x ⋅y ⋅z + 52⋅x⋅y⋅z + 48⎠⋅ℯ
       

      diff также можно назвать методом. Два способа вызова diff : точно такие же, и предоставлены только для удобства.

       >>> выр.разн(х, у, у, г, 4)
       3 2 ⎛ 3 3 3 2 2 2 ⎞ x⋅y⋅z
      х ⋅y ⋅⎝x ⋅y ⋅z + 14⋅x ⋅y ⋅z + 52⋅x⋅y⋅z + 48⎠⋅ℯ
       

      Чтобы создать неоцененную производную, используйте класс Derivative . Он имеет тот же синтаксис, что и diff .

       >>> производная = производная (выражение, x, y, y, z, 4)
      >>> производное
           7
          ∂ ⎛ х⋅y⋅z⎞
      ──────────⎝ℯ ⎠
        4 2
      ∂z ∂y ∂x
       

      Чтобы оценить неоцененный производный инструмент, используйте метод doit .

       >>> производное.doit()
       3 2 ⎛ 3 3 3 2 2 2 ⎞ x⋅y⋅z
      х ⋅y ⋅⎝x ⋅y ⋅z + 14⋅x ⋅y ⋅z + 52⋅x⋅y⋅z + 48⎠⋅ℯ
       

      Эти необработанные объекты полезны для задержки оценки производное или для целей печати. Они также используются, когда SymPy не уметь вычислять производную выражения (например, если оно содержит неопределенные функции, которые описаны в разделе Решение раздел «Дифференциальные уравнения»).

      Производные неуказанного порядка могут быть созданы с использованием кортежа (x, n) , где n – порядок производной относительно x .

       >>> m, n, a, b = символы ('m n a b')
      >>> выражение = (a*x + b)**m
      >>> выр.разн((х, п))
        н
       ∂ ⎛ м⎞
      ───⎝(а⋅х + б) ⎠
        н
      ∂х
       

      Интегралы

      Чтобы вычислить интеграл, используйте функцию интегрировать . Есть два вида интегралов, определенных и неопределённых. Чтобы вычислить неопределенный интеграл, то есть первообразная или примитивная, просто передайте переменную после выражение.

       >>> интегрировать(cos(x),x)
      грех (х)
       

      Обратите внимание, что SymPy не включает константу интегрирования. Если вы хотите, вы можете добавить его самостоятельно или перефразировать свою задачу в виде дифференциального уравнения и используйте dsolve для ее решения, что действительно добавляет константу (см. Решение дифференциальных уравнений).

      Чтобы вычислить определенный интеграл, передайте аргумент (интеграция_переменная, нижний_предел, верхний_предел) . Например, чтобы вычислить 9{2}}\, дх\, dy,\]

      до

       >>> интегрировать(exp(-x**2 - y**2), (x, -oo, oo), (y, -oo, oo))
      π
       

      Если интегрировать не удается вычислить интеграл, он возвращает невычисленное Интегральный объект .

       >>> выражение = интегрировать (х**х, х)
      >>> печать (выражение)
      Интеграл(х**х, х)
      >>> выражение
      ⌠
      ⎮ х
      ⎮ х дх
      ⌡
       

      То же, что и Производная , вы можете создать невычисленный интеграл, используя Интеграл . Чтобы позже оценить этот интеграл, позвоните по номеру или .

       >>> expr = Integral(log(x)**2, x)
      >>> выражение
      ⌠
      ⎮ 2
      ⎮ log (x) dx
      ⌡
      >>> expr.doit()
               2
      х⋅лог(х) - 2⋅х⋅лог(х) + 2⋅х
       

      интегрировать использует мощные алгоритмы, которые постоянно совершенствуются для вычислений как определенные, так и неопределенные интегралы, включая эвристическое сопоставление с образцом алгоритмы типа, частичная реализация алгоритма Риша и алгоритм, использующий G-функции Мейера, т.е. полезно для вычисления интегралов с точки зрения специальных функций, особенно определенные интегралы. Вот выборка некоторой степени интегрировать .

       >>> integ = Integral((x**4 + x**2*exp(x) - x**2 - 2*x*exp(x) - 2*x -
      ... exp(x))*exp(x)/((x - 1)**2*(x + 1)**2*(exp(x) + 1)), x)
      >>> целое число
      ⌠
      ⎮ ⎛ 4 2 х 2 х х⎞ х
      ⎮ ⎝x + x ⋅ℯ - x - 2⋅x⋅ℯ - 2⋅x - ℯ ⎠⋅ℯ
      ⎮ ──────────────────────────────────────── dx
      ⎮ 2 2 ⎛ х ⎞
      ⎮ (х - 1) ⋅(х + 1) ⋅⎝ℯ + 1⎠
      ⌡
      >>> integ. doit()
                       Икс
         ⎛ х ⎞ ℯ
      лог⎝ℯ + 1⎠ + ──────
                     2
                    х - 1
       
       >>> целое = Интеграл(sin(x**2),x)
      >>> целое число
      ⌠
      ⎮ ⎛ 2⎞
      ⎮ sin⎝x ⎠ dx
      ⌡
      >>> integ.doit()
               ⎛√2⋅х⎞
      3⋅√2⋅√π⋅S⎜────⎟⋅Γ(3/4)
               ⎝ √π ⎠
      ──────────────────────
             8⋅Г(7/4)
       
       >>> integ = Integral(x**y*exp(-x), (x, 0, oo))
      >>> целое число
      ∞
      ⌠
      ⎮ у -х
      ⎮ х ⋅ℯ дх
      ⌡
      0
      >>> integ.doit()
      ⎧ Γ(y + 1) при re(y) > -1
      ⎪
      ⎪∞
      ⎪⌠
      ⎨⎮ у -х
      ⎪⎮ x ⋅ℯ dx иначе
      ⎪⌡
      ⎪0
      ⎩
       

      В этом последнем примере возвращено выражение Кусочная , так как интеграл не сходится, если \(\Re(y) > 1.\)

      Пределы

      SymPy может вычислять символьные ограничения с помощью функции limit . Синтаксис для вычисления

      \[\lim_{x\to x_0} f(x)\]

      — это limit(f(x), x, x0) .

       >>> предел(грех(х)/х, х, 0)
      1
       

      предел следует использовать вместо подписки всякий раз, когда точка оценки является особенностью. Несмотря на то, что в SymPy есть объекты для представления \(\infty\), использование их для оценки не надежен, потому что они не следят за вещами как скорость роста. Кроме того, такие вещи, как \(\infty – \infty\) и \(\frac{\infty}{\infty}\) вернуть \(\mathrm{nan}\) (не число). Например

       >>> выражение = x**2/exp(x)
      >>> expr.subs(x, oo)
      нан
      >>> предел(выражение, х, оо)
      0
       

      Подобно Производная и Интеграл , предел имеет невычисленное аналог, Предел . Чтобы оценить его, используйте doit .

       >>> выражение = Предел((cos(x) - 1)/x, x, 0)
      >>> выражение
           ⎛cos(x) - 1⎞
       лим ⎜──────────⎟
      х─→0⁺⎝ х ⎠
      >>> expr.doit()
      0
       

      Чтобы оценить предел только с одной стороны, передайте '+' или '-' в качестве четвертого аргумент до 94\) опущены. Условия заказа можно создавать и управлять ими за пределами серии . Они автоматически поглощать члены более высокого порядка.

       >>> х + х**3 + х**6 + О(х**4)
           3 ⎛ 4⎞
      х + х + О⎝х ⎠
      >>> х*О(1)
      О (х)
       

      Если вам не нужен срок заказа, используйте метод removeO .

       >>> выр.серия(х, 0, 4).removeO()
       2
      Икс
      ── + х + 1
      2
       

      Нотация O поддерживает произвольные предельные точки (кроме 0):

       >>> exp(x - 6).series(x, x0=6)
                  2 3 4 5
           (х - 6) (х - 6) (х - 6) (х - 6) ⎛ 6 ⎞
      -5 + ──────── + ──────── + ──────── + ──────── + x + O⎝(x - 6) ; х → 6⎠
              2 6 24 120
       

      Конечные разности

      До сих пор мы рассматривали выражения с аналитическими производными и примитивные функции соответственно. Но что, если мы хотим иметь выражение для оценки производной кривой, для которой нам не хватает представление в закрытой форме, или для которого мы не знаем функционал значения для еще. Одним из подходов было бы использование конечной разности подход.

      Самый простой способ дифференцировать с помощью конечных разностей — использовать функция Different_finite :

       >>> f, g = символы ('f g', cls = функция)
      >>> Different_finite (f (x) * g (x))
      -f(x - 1/2)⋅g(x - 1/2) + f(x + 1/2)⋅g(x + 1/2)
       

      Если у вас уже есть производный экземпляр , вы можете использовать метод as_finite_difference для создания аппроксимаций производная в произвольном порядке:

       >>> f = Функция ('f')
      >>> dfdx = f(x). diff(x)
      >>> dfdx.as_finite_difference()
      -f(x - 1/2) + f(x + 1/2)
       

      здесь производная первого порядка была аппроксимирована вокруг x с использованием минимальное количество оцениваемых баллов (2 для производной 1-го порядка) равноудаленно, используя размер шага 1. Мы можем использовать произвольные шаги (возможно, содержащие символические выражения):

       >>> f = Функция ('f')
      >>> d2fdx2 = f(x).diff(x, 2)
      >>> ч = Символ('ч')
      >>> d2fdx2.as_finite_difference([-3*h,-h,2*h])
      f(-3⋅ч) f(-ч) 2⋅f(2⋅ч)
      ─────── - ───── + ────────
           2 2 2
        5⋅ч 3⋅ч 15⋅ч
       

      Если вы просто заинтересованы в оценке веса, вы можете сделать это вручную:

       >>> final_diff_weights(2, [-3, -1, 2], 0)[-1][-1]
      [1/5, -1/3, 2/15]
       

      обратите внимание, что нам нужен только последний элемент в последнем подсписке возвращено из final_diff_weights . Причина этого в том, что функция также генерирует веса для младших производных и с использованием меньшего количества точек (см. документацию final_diff_weights Больше подробностей).

      Если использование final_diff_weights напрямую выглядит сложным, то as_finite_difference метод производных экземпляров недостаточно гибок, вы можете использовать apply_finite_diff , который принимает порядка , x_list , y_list и x0 в качестве параметров:

       >>> x_list = [-3, 1, 2]
      >>> y_list = символы ('a b c')
      >>> apply_finite_diff(1, x_list, y_list, 0)
        3⋅а б 2⋅с
      - ─── - ─ + ───
         20 4 5
       

      3.2 Производная как функция. Исчисление, том 1

      Цели обучения

      • 3.2.1 Определите производную функцию данной функции.
      • 3.2.2 График производной функции по графику заданной функции.
      • 3.2.3 Укажите связь между производными и непрерывностью.
      • 3.2.4 Назовите три условия, при которых функция не имеет производной.
      • 3.2.5 Объясните значение производной высшего порядка.

      Как мы видели, производная функции в данной точке дает нам скорость изменения или наклон касательной к функции в этой точке. Если мы продифференцируем функцию положения в данный момент времени, мы получим скорость в этот момент. Кажется разумным заключить, что знание производной функции в каждой точке даст ценную информацию о поведении функции. Однако процесс нахождения производной даже при нескольких значениях с использованием методов из предыдущего раздела быстро стал бы довольно утомительным. В этом разделе мы определяем производную функцию и изучаем процесс ее нахождения.

      Производные функции

      Функция производной дает производную функции в каждой точке области определения исходной функции, для которой определена производная. Мы можем формально определить производную функцию следующим образом.

      Определение

      Пусть ff — функция. Производная функция, обозначаемая через f′,f′, представляет собой функцию, область определения которой состоит из таких значений xx, что существует следующий предел:

      f′(x)=limh→0f(x+h)−f(x )h.f′(x)=limh→0f(x+h)−f(x)h.

      (3.9)

      Функция f(x)f(x) называется дифференцируемой в п.в., если f′(a)f′(a) существует. В более общем смысле функция называется дифференцируемой на SS, если она дифференцируема в каждой точке открытого множества S, S, а дифференцируемая функция — это функция, в которой f′(x)f′(x) существует в своей области определения.

      В следующих нескольких примерах мы используем уравнение 3.9 для нахождения производной функции.

      Пример 3.11

      Нахождение производной функции квадратного корня

      Найдите производную f(x)=x.f(x)=x.

      Решение

      Начните непосредственно с определения функции производной. Используйте уравнение 3. 1.

      f′(x)=limh→0x+h−xhЗаменитьef(x+h)=x+handf(x)=xintof′(x)=limh→0f(x+h)−f(x)h.= limh→0x+h−xh·x+h+xx+h+xУмножить числитель и знаменатель наx+h+x без распределения в знаменателе.=limh→0hh(x+h+x)Умножить числители и упростить.=limh→01 (x+h+x)Отменить h.=12xВычислить предел.f′(x)=limh→0x+h−xhЗаменитьef(x+h)=x+handf(x)=xintof′(x)=limh→0f (x+h)−f(x)h.=limh→0x+h−xh·x+h+xx+h+xУмножить числитель и знаменатель наx+h+x без распределения в знаменателе.=limh→0hh(x+h +x)Умножить числители и упростить.=limh→01(x+h+x)Отменить h.=12xВычислить предел.

      Пример 3.12

      Нахождение производной квадратичной функции

      Нахождение производной функции f(x)=x2−2x.f(x)=x2−2x.

      Решение

      Выполните ту же процедуру здесь, но без умножения на сопряженное.

      f′(x)=limh→0((x+h)2−2(x+h))−(x2−2x)hПодставитьef(x+h)=(x+h)2−2(x+ h)иf(x)=x2−2xintof′(x)=limh→0f(x+h)−f(x)h.=limh→0x2+2xh+h3−2x−2h−x2+2xhExpand(x+h )2−2(x+h). =limh→02xh−2h+h3hУпростить.=limh→0h(2x−2+h)hМножитель hвыпадает из числителя.=limh→0(2x−2+h)Отменить общий множитель ofh.=2x−2Вычислить предел.f′(x)=limh→0((x+h)2−2(x+h))−(x2−2x)hПодставитьef(x+h)=(x+h )2−2(x+h) и f(x)=x2−2xintof′(x)=limh→0f(x+h)−f(x)h.=limh→0x2+2xh+h3−2x−2h− x2+2xhExpand(x+h)2−2(x+h).=limh→02xh−2h+h3hУпростить.=limh→0h(2x−2+h)hМножитель hиз числителя.=limh→0(2x−2 +h)Отменить общий множитель h.=2x−2Вычислить предел.

      Контрольно-пропускной пункт 3,6

      Найдите производную f(x)=x2.f(x)=x2.

      Мы используем различные обозначения для выражения производной функции. В примере 3.12 мы показали, что если f(x)=x2−2x,f(x)=x2−2x, то f′(x)=2x−2.f′(x)=2x−2. Если бы мы представили эту функцию в виде y=x2−2x,y=x2−2x, мы могли бы выразить производную как y′=2x−2y′=2x−2 или dydx=2x−2.dydx=2x− 2. Мы могли бы передать ту же информацию, написав ddx(x2−2x)=2x−2.ddx(x2−2x)=2x−2. Таким образом, для функции y=f(x),y=f(x) каждое из следующих обозначений представляет собой производную от f(x):f(x):

      f′(x),dydx,y′,ddx(f(x)). f′(x),dydx,y′,ddx(f(x)).

      Вместо f′(a)f′(a) мы также можем использовать dydx|x=adydx|x=a Использование нотации dydxdydx (называемой нотацией Лейбница) довольно распространено в технике и физике. Чтобы лучше понять эти обозначения, вспомним, что производная функции в точке — это предел наклона секущих по мере приближения секущих к касательной. Наклоны этих секущих часто выражаются в виде ΔyΔxΔyΔx, где ΔyΔy — разность значений yy, соответствующая разнице значений xx, которая выражается как ΔxΔx (рис. 3.11). Таким образом, производная, которую можно рассматривать как мгновенную скорость изменения yy по отношению к x,x, выражается как

      dydx=limΔx→0ΔyΔx.dydx=limΔx→0ΔyΔx.

      Рисунок 3.11 Производная выражается как dydx=limΔx→0ΔyΔx.dydx=limΔx→0ΔyΔx.

      График производной

      Мы уже обсуждали, как построить график функции, поэтому, имея уравнение функции или уравнение производной функции, мы могли бы построить график. Учитывая оба, мы ожидаем увидеть соответствие между графиками этих двух функций, поскольку f′(x)f′(x) дает скорость изменения функции f(x)f(x) (или наклон кривой). касательная к f(x)).f(x)).

      В примере 3.11 мы обнаружили, что для f(x)=x,f′(x)=1/2x.f(x)=x,f′(x)=1/2x. Если мы изобразим эти функции на тех же осях, как на рис. 3.12, мы сможем использовать графики, чтобы понять взаимосвязь между этими двумя функциями. Во-первых, мы замечаем, что f(x)f(x) возрастает по всей своей области, а это означает, что наклоны ее касательных во всех точках положительны. Следовательно, мы ожидаем, что f′(x)>0f′(x)>0 для всех значений xx в его области определения. Кроме того, по мере увеличения xx наклоны касательных линий к f(x)f(x) уменьшаются, и мы ожидаем увидеть соответствующее уменьшение f′(x).f′(x). Заметим также, что f′(0)f′(0) не определено и что limx→0+f′(x)=+∞, limx→0+f′(x)=+∞, что соответствует вертикальной касательной к f(x)f(x) при 0,0.

      Рисунок 3.12 Производная f′(x)f′(x) везде положительна, поскольку функция f(x)f(x) возрастает.

      В примере 3.12 мы нашли, что для f(x)=x2−2x,f′(x)=2x−2.f(x)=x2−2x,f′(x)=2x−2. Графики этих функций показаны на рис. 3.13. Обратите внимание, что f(x)f(x) убывает при x<1.x<1. Для этих же значений x,f′(x)<0.x,f′(x)<0. Для значений x>1,f(x)x>1,f(x) возрастает, а f′(x)>0.f′(x)>0. Кроме того, f(x)f(x) имеет горизонтальную касательную в точке x=1x=1 и f′(1)=0.f′(1)=0.

      Рисунок 3.13 Производная f′(x)<0f′(x)<0, где функция f(x)f(x) убывающая и f′(x)>0f′(x)>0, где f(x)f(x) ) растет. Производная равна нулю там, где функция имеет горизонтальный тангенс.

      Пример 3.13

      Набросок производной с использованием функции

      Используйте следующий график функции f(x)f(x), чтобы нарисовать график функции f′(x).f′(x).

      Решение

      Решение показано на следующем графике. Заметим, что f(x)f(x) возрастает и f′(x)>0f′(x)>0 на (–2,3).(–2,3). Кроме того, f(x)f(x) убывает и f′(x)<0f′(x)<0 на (−∞,−2)(−∞,−2) и на (3,+∞). (3,+∞). Также обратите внимание, что f(x)f(x) имеет горизонтальные касательные в точках –2–2 и 3,3, а также f′(−2)=0f′(−2)=0 и f′(3)=0. f ′(3)=0.

      Контрольно-пропускной пункт 3,7

      Нарисуйте график функции f(x)=x2−4.f(x)=x2−4. На каком интервале находится график функции f′(x)f′(x) над осью xx?

      Производные и непрерывность

      Теперь, когда мы можем построить график производной, давайте рассмотрим поведение графиков. Сначала рассмотрим связь между дифференцируемостью и непрерывностью. Мы увидим, что если функция дифференцируема в точке, она должна быть там непрерывной; однако функция, непрерывная в точке, не обязательно должна быть дифференцируемой в этой точке. На самом деле функция может быть непрерывной в точке и не быть дифференцируемой в этой точке по одной из нескольких причин.

      Теорема 3.1

      Дифференцируемость подразумевает непрерывность

      Пусть f(x)f(x) — функция, aa — в ее области определения. Если f(x)f(x) дифференцируема в точках а, а, то функция ff непрерывна в точках а.

      Доказательство

      Если f(x)f(x) дифференцируема в точках a,a, то f′(a)f′(a) существует и

      f′(a)=limx→af(x)−f(a)x−a. f′(a)=limx→af(x)−f(a)x−a.

      Мы хотим показать, что функция f(x)f(x) непрерывна в точке aa, показав, что limx→af(x)=f(a).limx→af(x)=f(a). Таким образом,

      limx→af(x)=limx→a(f(x)−f(a)+f(a))=limx→a(f(x)−f(a)x−a·(x−a) +f(a)) Умножить и разделить f(x)−f(a)наx−a.=(limx→af(x)−f(a)x−a)·(limx→a(x−a))+ limx→af(a)=f′(a)·0+f(a)=f(a).limx→af(x)=limx→a(f(x)−f(a)+f(a) )=limx→a(f(x)−f(a)x−a·(x−a)+f(a)) Умножить и разделить f(x)−f(a)наx−a.=(limx→af (x)−f(a)x−a)·(limx→a(x−a))+limx→af(a)=f′(a)·0+f(a)=f(a).

      Следовательно, поскольку f(a)f(a) определено и limx→af(x)=f(a),limx→af(x)=f(a), мы заключаем, что ff непрерывен в п.в.

      Мы только что доказали, что дифференцируемость влечет непрерывность, но теперь мы рассмотрим, влечет ли непрерывность дифференцируемость. Чтобы найти ответ на этот вопрос, мы исследуем функцию f(x)=|x|.f(x)=|x|. Эта функция всюду непрерывна; однако f′(0)f′(0) не определено. Это наблюдение приводит нас к мысли, что непрерывность не влечет дифференцируемости. Давайте исследовать дальше. Для f(x)=|x|,f(x)=|x|,

      f′(0)=limx→0f(x)−f(0)x−0=limx→0|x|−|0|x−0=limx→0|x|x.f′(0)=limx→ 0f(x)−f(0)x−0=limx→0|x|−|0|x−0=limx→0|x|x.

      Это ограничение не существует, так как

      limx→0−|x|x=−1и limx→0+|x|x=1.limx→0−|x|x=−1иlimx→0+|x|x=1.

      См. рис. 3.14.

      Рисунок 3.14 Функция f(x)=|x|f(x)=|x| непрерывна в точке 00, но не дифференцируема в точке 0,0.

      Рассмотрим некоторые дополнительные ситуации, в которых непрерывная функция не может быть дифференцируемой. Рассмотрим функцию f(x)=x3:f(x)=x3:

      f′(0)=limx→0x3−0x−0=limx→01×23=+∞.f′(0)=limx→0x3−0x−0=limx→01×23=+∞.

      Таким образом, f′(0)f′(0) не существует. Беглый взгляд на график f(x)=x3f(x)=x3 проясняет ситуацию. Функция имеет вертикальную касательную в точке 00 (рис. 3.15).

      Рисунок 3.15 Функция f(x)=x3f(x)=x3 имеет вертикальную касательную в точке x=0.x=0. Он непрерывен в точке 00, но не дифференцируем в точке 0,0.

      Функция f(x)={xsin(1x)ifx≠00ifx=0f(x)={xsin(1x)ifx≠00ifx=0 также имеет производную, которая демонстрирует интересное поведение при 0,0. Мы видим, что

      f′(0)=limx→0xsin(1/x)−0x−0=limx→0sin(1x).f′(0)=limx→0xsin(1/x)−0x−0=limx→0sin( 1х).

      Этого предела не существует, в основном потому, что наклоны секущих постоянно меняют направление по мере приближения к нулю (рис. 3.16).

      Рисунок 3.16 Функция f(x)={xsin(1x)ifx≠00ifx=0f(x)={xsin(1x)ifx≠00ifx=0 не дифференцируема при 0,0.

      Итого:

      1. Заметим, что если функция не непрерывна, то она не может быть дифференцируемой, так как каждая дифференцируемая функция должна быть непрерывной. Однако, если функция непрерывна, она может не быть дифференцируемой.
      2. Мы видели, что f(x)=|x|f(x)=|x| не мог быть дифференцируем в 00, потому что предел наклонов касательных слева и справа не был одинаковым. Визуально это привело к появлению острого угла на графике функции в точке 0. 0. Отсюда заключаем, что для того, чтобы быть дифференцируемой в точке, функция должна быть «гладкой» в этой точке.
      3. Как мы видели на примере f(x)=x3,f(x)=x3, функция не может быть дифференцируемой в точке, где есть вертикальная касательная.
      4. Как мы видели с f(x)={xsin(1x)ifx≠00ifx=0f(x)={xsin(1x)ifx≠00ifx=0 функция может не быть дифференцируемой в точке и более сложными способами .

      Пример 3.14

      Кусочная функция, которая является непрерывной и дифференцируемой

      Компания по производству игрушек хочет разработать траекторию для игрушечной машинки, которая начинается с параболической кривой, а затем переходит в прямую линию (рис. 3.17). Функция, описывающая дорожку, должна иметь вид −10, где xx и f(x)f(x) указаны в дюймах. Чтобы автомобиль двигался плавно по трассе, функция f(x)f(x) должна быть одновременно непрерывной и дифференцируемой при −10,−10. Найдите значения bb и cc, которые делают f(x)f(x) непрерывным и дифференцируемым.

      Рисунок 3.17 Чтобы автомобиль двигался плавно по трассе, функция должна быть одновременно и непрерывной, и дифференцируемой.

      Решение

      Чтобы функция была непрерывной при x=−10,limx→−10−f(x)=f(−10).x=−10,limx→−10−f(x)=f(−10). Таким образом, поскольку

      limx→−10−f(x)=110(−10)2−10b+c=10−10b+climx→−10−f(x)=110(−10)2−10b+c =10−10b+c

      и f(−10)=5,f(−10)=5, мы должны иметь 10−10b+c=5,10−10b+c=5. Эквивалентно, мы имеем c=10b−5.c=10b−5.

      Чтобы функция была дифференцируемой при −10, −10,

      f′(-10)=limx→−10f(x)−f(−10)x+10f′(-10)=limx→−10f(x)−f(−10)x+10

      обязательно существует. Поскольку f(x)f(x) определяется по разным правилам справа и слева, мы должны вычислить этот предел справа и слева, а затем приравнять их друг другу:

      limx→−10−f( x)−f(−10)x+10=limx→−10−110×2+bx+c−5x+10=limx→−10−110×2+bx+(10b−5)−5x+10Substitutec=10b−5.= limx→−10−x2−100+10bx+100b10(x+10)=limx→−10−(x+10)(x−10+10b)10(x+10) Фактор по группировке. =b−2. limx→−10−f(x)−f(−10)x+10=limx→−10−110×2+bx+c−5x+10=limx→−10−110×2+bx+(10b−5)−5x+ 10Подставитьc=10b−5.=limx→−10−x2−100+10bx+100b10(x+10)=limx→−10−(x+10)(x−10+10b)10(x+10)Множитель на группировка.=b−2.

      У нас также есть

      limx→−10+f(x)−f(−10)x+10=limx→−10+−14x+52−5x+10=limx→−10+−(x+10 )4(x+10)=−14.limx→−10+f(x)−f(−10)x+10=limx→−10+−14x+52−5x+10=limx→−10+− (х+10)4(х+10)=-14.

      Это дает нам b−2=−14.b−2=−14. Таким образом, b=74b=74 и c=10(74)−5=252.c=10(74)−5=252.

      Контрольно-пропускной пункт 3,8

      Найдите значения aa и bb, которые делают f(x)={ax+bifx<3x2ifx≥3f(x)={ax+bifx<3x2ifx≥3 непрерывными и дифференцируемыми в 3.3.

      Производные высшего порядка

      Производная функции сама по себе является функцией, поэтому мы можем найти производную производной. Например, производная функции положения — это скорость изменения положения или скорость. Производная скорости — это скорость изменения скорости, то есть ускорение. Новая функция, полученная дифференцированием производной, называется второй производной. Кроме того, мы можем продолжать брать производные, чтобы получить третью производную, четвертую производную и так далее. В совокупности они называются производными более высокого порядка. Обозначение производных высшего порядка от y=f(x)y=f(x) может быть выражено в любой из следующих форм:

      f″(x),f‴(x),f(4)(x),…,f(n)(x)f″(x),f‴(x),f(4)(x), …,f(n)(x)

      y″(x),y‴(x),y(4)(x),…,y(n)(x)y″(x),y‴(x),y(4)(x), …,у(п)(х)

      d2ydx2,d3ydx3,d4ydx4,…,dnydxn.d2ydx2,d3ydx3,d4ydx4,…,dnydxn.

      Интересно отметить, что нотация для d2ydx2d2ydx2 может рассматриваться как попытка выразить ddx(dydx)ddx(dydx) более компактно. Аналогично, ddx(ddx(dydx))=ddx(d2ydx2)=d3ydx3.ddx(ddx(dydx))=ddx(d2ydx2)=d3ydx3.

      Пример 3.15

      Нахождение второй производной

      Для f(x)=2×2−3x+1,f(x)=2×2−3x+1 найти f″(x).f″(x).

      Решение

      Сначала найдите f′(x). f′(x).

      f′(x)=limh→0(2(x+h)2−3(x+h)+1)−(2×2−3x+1)hПодставить f(x)=2×2−3x+1andf(x+ h)=2(x+h)2−3(x+h)+1intof′(x)=limh→0f(x+h)−f(x)h.=limh→04xh+2h3−3hhУпростим числитель. =limh→0(4x+2h−3) Вынести в числитель и сократить в знаменателе.=4x−3 Возьмем предел. f′(x)=limh→0(2(x+h)2−3(x+ h)+1)−(2×2−3x+1)hПодставитьef(x)=2×2−3x+1andf(x+h)=2(x+h)2−3(x+h)+1intof′(x)= limh→0f(x+h)−f(x)h.=limh→04xh+2h3−3hhУпростить числитель.=limh→0(4x+2h−3)Вынести из числителя в множитель и сократить в знаменателе.=4x− 3Возьмите лимит.

      Затем найдите f″(x)f″(x), взяв производную от f′(x)=4x−3.f′(x)=4x−3.

      f″(x)=limh→0f′(x+h)−f′(x)hUsef′(x)=limh→0f(x+h)−f(x)hс f′(x) на месте off( x).=limh→0(4(x+h)−3)−(4x−3)hПодставьте f′(x+h)=4(x+h)−3 и f′(x)=4x−3.=limh →04Упростить.=4Возьмем предел. (x) вместо off(x).=limh→0(4(x+h)−3)−(4x−3)hПодставить f′(x+h)=4(x+h)−3andf′(x)= 4x−3.=limh→04Упростить.=4Возьмем предел.

      Контрольно-пропускной пункт 3,9

      Найдите f″(x)f″(x) для f(x)=x2. f(x)=x2.

      Пример 3.16

      Определение ускорения

      Положение частицы вдоль координатной оси в момент времени tt (в секундах) определяется выражением s(t)=3t2−4t+1s(t)=3t2−4t+1 (в метрах). Найдите функцию, описывающую его ускорение в момент времени t.t.

      Решение

      Поскольку v(t)=s′(t)v(t)=s′(t) и a(t)=v′(t)=s″(t), a(t)=v′(t) =s″(t), начнем с нахождения производной от s(t):s(t):

      s′(t)=limh→0s(t+h)−s(t)h=limh→03 (t+h)2−4(t+h)+1−(3t2−4t+1)h=6t−4.s′(t)=limh→0s(t+h)−s(t)h= limh→03(t+h)2−4(t+h)+1−(3t2−4t+1)h=6t−4.

      Далее,

      s″(t)=limh→0s′(t+h)−s′(t)h=limh→06(t+h)−4−(6t−4)h=6.s ″(t)=limh→0s′(t+h)−s′(t)h=limh→06(t+h)−4−(6t−4)h=6.

      Таким образом, a=6м/с2.a=6м/с2.

      Контрольно-пропускной пункт 3.10

      Для s(t)=t3,s(t)=t3 найти a(t).a(t).

      Раздел 3.2 Упражнения

      В следующих упражнениях используйте определение производной, чтобы найти f′(x). f′(x).

      54.

      f(x)=6f(x)=6

      55.

      f(x)=2−3xf(x)=2−3x

      56.

      f(x)=2×7+1f(x)=2×7+1

      57.

      f(x)=4x2f(x)=4×2

      58.

      f(x)=5x−x2f(x)=5x−x2

      59.

      f(x)=2xf(x)=2x

      60.

      f(x)=x−6f(x)=x−6

      61.

      f(x)=9xf(x)=9x

      62.

      f(x)=x+1xf(x)=x+1x

      63.

      f(x)=1xf(x)=1x

      В следующих упражнениях используйте график y=f(x)y=f(x), чтобы начертить график его производной f′(x).f′(x).

      64.

      65.

      66.

      67.

      Для следующих упражнений данный предел представляет собой производную функции y=f(x)y=f(x) при x=a.x=a. Найти f(x)f(x) и п.в.

      68.

      limh→0(1+h)2/3−1hlimh→0(1+h)2/3−1h

      69.

      limh→0[3(2+h)2+2]−14hlimh→0[3(2+h)2+2]−14h

      70.

      limh→0cos(π+h)+1hlimh→0cos(π+h)+1h

      71.

      limh→0(2+h)4−16hlimh→0(2+h)4−16h

      72.

      limh→0[2(3+h)2−(3+h)]−15hlimh→0[2(3+h)2−(3+h)]−15h

      73.

      limh→0eh−1hlimh→0eh−1h

      Для следующих функций,

      1. эскиз графика и
      2. используют определение производной, чтобы показать, что функция не дифференцируема при x=1.x=1.

      74.

      f(x)={2x,0≤x≤13x−1,x>1f(x)={2x,0≤x≤13x−1,x>1

      75.

      f(x)={3,x<13x,x≥1f(x)={3,x<13x,x≥1

      76.

      f(x)={−x2+2,x≤1x,x>1f(x)={−x2+2,x≤1x,x>1

      77.

      f(x)={2x,x≤12x,x>1f(x)={2x,x≤12x,x>1

      Для следующих графиков

      1. определить, для каких значений x=ax=a существует limx→af(x)limx→af(x), но ff не является непрерывным при x=a,x=a, и
      2. определить, при каких значениях x=ax=a функция непрерывна, но не дифференцируема при x=a.x=a.

      78.

      79.

      80.

      Используйте график для оценки a. f'(-0,5),f'(-0,5), б. f′(0),f′(0), в. f′(1),f′(1), д. f′(2),f′(2) и e. f′(3),f′(3), если он существует.

      Для следующих функций используйте f″(x)=limh→0f′(x+h)−f′(x)hf″(x)=limh→0f′(x+h)−f′(x)h чтобы найти f″(x).f″(x).

      81.

      f(x)=2−3xf(x)=2−3x

      82.

      f(x)=4x2f(x)=4×2

      83.

      f(x)=x+1xf(x)=x+1x

      В следующих упражнениях используйте калькулятор для построения графика f(x).f(x). Определите функцию f′(x),f′(x), затем используйте калькулятор для построения графика f′(x).f′(x).

      84.

      [Т] f(x)=−5xf(x)=−5x

      85.

      [Т] f(x)=3×2+2x+4.f(x)=3×2+2x+4.

      86.

      [Т] f(x)=x+3xf(x)=x+3x

      87.

      [Т] f(x)=12xf(x)=12x

      88.

      [Т] f(x)=1+x+1xf(x)=1+x+1x

      89.

      [Т] f(x)=x3+1f(x)=x3+1

      Для следующих упражнений опишите, что представляют два выражения в терминах каждой из данных ситуаций. Обязательно укажите единицы измерения.

      1. f(x+h)−f(x)hf(x+h)−f(x)h
      2. f′(x)=limh→0f(x+h)−f(x)hf′(x)=limh→0f(x+h)−f(x)h

      90.

      P(x)P(x) обозначает население города в момент времени xx в годах.

      91.

      C(x)C(x) обозначает общую сумму денег (в тысячах долларов), потраченную на концессии xx клиентов в парке развлечений.

      92.

      R(x)R(x) обозначает общую стоимость (в тысячах долларов) изготовления xx радиочасов.

      93.

      g(x)g(x) обозначает оценку (в процентах), полученную за тест при xx часах обучения.

      94.

      B(x)B(x) обозначает стоимость (в долларах) учебника по социологии в университетских книжных магазинах США в xx лет, начиная с 1990.1990.

      95.

      p(x)p(x) обозначает атмосферное давление в торрах на высоте xx футов.

      96.

      Нарисуйте график функции y=f(x)y=f(x) со всеми следующими свойствами:

      1. f′(x)>0f′(x)>0 для −2≤x<1−2≤x<1
      2. f'(2)=0f'(2)=0
      3. f′(x)>0f′(x)>0 для x>2x>2
      4. f(2)=2f(2)=2 и f(0)=1f(0)=1
      5. limx→−∞f(x)=0limx→−∞f(x)=0 и limx→∞f(x)=∞limx→∞f(x)=∞
      6. f′(1)f′(1) не существует.

      97.

      Предположим, что температура TT в градусах Фаренгейта на высоте xx в футах над землей равна y=T(x).y=T(x).

      1. Дайте физическую интерпретацию с единицами измерения T′(x).T′(x).
      2. Если мы знаем, что T′(1000)=−0,1,T′(1000)=−0,1, объясни физический смысл.

      98.

      Предположим, что общая прибыль компании составляет y=P(x)y=P(x) тысяч долларов при продаже xx единиц товара.

      1. Что измеряет P(b)−P(a)b−aP(b)−P(a)b−a для 0
      2. Что измеряет P′(x)P′(x) и в каких единицах?
      3. Предположим, что P'(30)=5, P'(30)=5, как приблизительно изменится прибыль, если количество проданных товаров увеличится с 30 до 31?30 до 31?

      99.

      График на следующем рисунке моделирует количество людей N(t)N(t), заболевших гриппом tt недель после его первоначальной вспышки в городе с населением 50 000 50 000 жителей.

      1. Опишите, что представляет собой N'(t)N'(t) и как она ведет себя при увеличении tt.
      2. Что эта производная говорит нам о том, как этот город пострадал от вспышки гриппа?

      Для следующих упражнений используйте следующую таблицу, в которой показана высота hh ракеты Saturn VV для миссии Apollo 1111 через tt секунд после запуска.

      Время (секунды) Высота (метры)
      00 00
      11 22
      22 44
      33 1313
      44 2525
      55 3232

      100.

      Каков физический смысл h'(t)?h'(t)? Что такое единицы?

      101.

      [T] Постройте таблицу значений для h'(t)h'(t) и отобразите h(t)h(t) и h'(t)h'(t) на одном графике. ( Подсказка: для внутренних точек, оцените как левый предел, так и правый предел и усредните их. Внутренняя точка интервала I — это элемент I, который не является конечной точкой I.)

      102.

      [T] Наилучшая линейная аппроксимация данных дается формулой H(t)=7,229t−4,905, H(t)=7,229t−4,905, где HH — высота ракеты (в метрах) и tt — время, прошедшее с момента взлета. Из этого уравнения определите H′(t).H′(t). График H(t)H(t) с заданными данными и на отдельной координатной плоскости график H′(t).H′(t).

      103.

      [T] Наилучшее квадратичное соответствие данным определяется выражением G(t)=1,429t2+0,0857t−0,1429,G(t)=1,429t2+0,0857t−0,1429, где GG — высота ракеты (в метрах) и tt — время, прошедшее с момента взлета.

      Оставить комментарий